134
SURGERY 1. A man sustained severe blunt injuries in a car crash twelve hours ago. The activity of this hormone is now expected to be decreased in this patient: A. insulin B. cortisol C. epinephrine D. aldosterone (Classification - Application; Source – Schwartz’s Textbook of Surgery (8 th edition): pp. 25-29) 2. A young woman is debilitated by pruritus and burning sensation from her presternal keloid. The recommended initial treatment is: A. topical application of silicone sheets B. intralesional corticosteroid injection C. surgical excision D. low-dose radiation (Classification - Recall; Source – Schwartz’s Textbook of Surgery (8 th edition): pp. 241-242) 3. A patient is placed on NPO in preparation for an elective major abdominal surgery. The body’s preferred initial fuel source during the fasting state is: A. hepatic glycogen B. skeletal muscle glycogen C. muscle protein D. body fat (Classification - Recall; Source – Schwartz’s Textbook of Surgery (8 th edition): p. 24) 4. A 48-year-old woman has prolonged ileus after surgery for an obstructed duodenal ulcer. The problem is probably due to: A. hypochloremia B. hypocalcemia C. hypomagnesemia D. hypokalemia (Classification - Application; Source – Schwartz’s Textbook of Surgery (8 th edition): pp. 48-49) 5. A man with chronic prepyloric ulcer appears weak after repeated bouts of non-bilious vomiting over the past 3 days. Fluid therapy should be started using: 1

Surgery- board exam questions

Embed Size (px)

Citation preview

Page 1: Surgery- board exam questions

SURGERY

1. A man sustained severe blunt injuries in a car crash twelve hours ago. The activity of this hormone is now expected to be decreased in this patient:

A. insulin B. cortisolC. epinephrineD. aldosterone

(Classification - Application; Source – Schwartz’s Textbook of Surgery (8th edition): pp. 25-29)

2. A young woman is debilitated by pruritus and burning sensation from her presternal keloid. The recommended initial treatment is:A. topical application of silicone sheetsB. intralesional corticosteroid injectionC. surgical excisionD. low-dose radiation

(Classification - Recall; Source – Schwartz’s Textbook of Surgery (8th edition): pp. 241-242)

3. A patient is placed on NPO in preparation for an elective major abdominal surgery. The body’s preferred initial fuel source during the fasting state is: A. hepatic glycogenB. skeletal muscle glycogenC. muscle proteinD. body fat

(Classification - Recall; Source – Schwartz’s Textbook of Surgery (8th edition): p. 24)

4. A 48-year-old woman has prolonged ileus after surgery for an obstructed duodenal ulcer. The problem is probably due to:A. hypochloremiaB. hypocalcemiaC. hypomagnesemiaD. hypokalemia

(Classification - Application; Source – Schwartz’s Textbook of Surgery (8th edition): pp. 48-49)

5. A man with chronic prepyloric ulcer appears weak after repeated bouts of non-bilious vomiting over the past 3 days. Fluid therapy should be started using:A. Normosol MB. Lactated Ringer’s solutionC. Normal saline solutionD. Hypertonic saline solution

(Classification - Application; Source – Schwartz’s Textbook of Surgery (8th edition): pp. 51-52)

6. A multiply injured patient has persistently low urine output. The oliguria is most likely due to prerenal failure rather than acute tubular necrosis if the tests reveal:A. low urine specific gravityB. low urinary excretion of sodium

1

Page 2: Surgery- board exam questions

C. low BUN/creatinine ratioD. low creatinine clearance

(Classification - Application; Source – Schwartz’s Textbook of Surgery (8th edition): pp. 348-349)

7. A woman admitted to the ICU for severe acute pancreatitis begins to manifest paresthesia of the face and extremities, muscle cramps, and a positive Chvostek’s sign. These are probably due to:A. hypocalcemiaB. hypokalemiaC. hypomagnesemiaD. hypophosphatemia

(Classification - Application; Source – Schwartz’s Textbook of Surgery (8th edition): p. 50)

8. Increased nutritional support appears to aggravate tachypnea in a septic patient with respiratory failure. Lessening the amount of this substrate may alleviate the problem:A. carbohydrateB. fat emulsionC. essential amino acidsD. non-essential amino acids

(Classification - Recall; Source – Schwartz’s Textbook of Surgery (8th edition): pp. 28-29)

9. Administration of this amino acid is expected to be beneficial to a patient suffering from damage to intestinal mucosa due to adjuvant chemotherapy for esophageal cancer:A. arginineB. valineC. leucineD. glutamine

(Classification - Recall; Source – Schwartz’s Textbook of Surgery (8th edition): p. 38)10. Early supplementation of this vitamin is recommended to promote wound

repair in a patient with extensive second-degree flame burns:A. AB. CC. DD. E

(Classification - Application; Source – Schwartz’s Textbook of Surgery (8th edition): p. 210)

11. A diabetic 41-year-old man is admitted for necrotizing fasciitis in the perineum. A high dose of this antibiotic is generally included in the initial antimicrobial therapy because of concern for clostridial pathogens: A. metronidazoleB. vancomycinC. penicillin GD. aminoglycoside

(Classification - Recall; Source – Schwartz’s Textbook of Surgery (8th edition): p. 122)

12. Transfusion of properly cross matched blood is begun on a man admitted for massive bleeding from erosive gastritis and thirty minutes later, he develops urticaria and fever. This should be administered to the patient:A. antihistamine

2

Page 3: Surgery- board exam questions

B. mannitolC. furosemideD. sodium bicarbonate

(Classification - Application; Source – Schwartz’s Textbook of Surgery (8th edition): pp. 79-80)

13. A patient under anticoagulation therapy using warfarin, who is set to undergo surgery for acute cholecystitis, has decreased prothrombin concentration. Warfarin can be reversed by parenteral dose of:A. protamine sulfateB. vitamin KC. EACAD. hydroxyurea

(Classification - Recall; Source – Schwartz’s Textbook of Surgery (8th edition): pp. 73-74)

14. Arterial blood gas analysis is performed on a patient just admitted with a diagnosis of severe acute pancreatitis. This reveals a pH of 7.30 and low levels of bicarbonate and pCO2. The most urgent part of management is: A. volume resuscitationB. intravenous bicarbonateC. calcium infusionD. mechanical ventilation

(Classification - Application; Source – Schwartz’s Textbook of Surgery (8th edition): pp. 50-51)

15. Seven days after surgery for a perforated appendicitis, the primarily closed incision is noted to be erythematous, slightly swollen and tender. The appropriate treatment is:A. local heat therapyB. topical antibiotics C. new systemic antibioticsD. incision and drainage

(Classification - Application; Source – Schwartz’s Textbook of Surgery (8th edition): pp. 119-120)

16. A man is brought to the E.R. with blood spurting from a hacking wound in the in the distal right thigh. He is alert and has a systolic BP of 100 mmHg. What is the initial management step?A. apply direct pressure on the wound with sterile gauzeB. apply digital pressure on proximal femoral artery C. apply a thigh tourniquet above the woundD. open the wound and clamp the bleeders

(Classification - Application; Source – Schwartz’s Textbook of Surgery (8th edition): p. 74)

17. A young man presents to the E.R. with a stab wound in the left chest. Examination reveals subcutaneous emphysema and absent breath sounds on the left chest; the trachea is shifted to the right. What is the probable diagnosis?A. massive hemothoraxB. tension pneumothoraxC. cardiac tamponadeD. flail chest

(Classification - Application; Source – Schwartz’s Textbook of Surgery (8th edition): p. 131)

3

Page 4: Surgery- board exam questions

18. A 58-year-old woman on NSAID therapy for arthritis has a 6-hour history of epigastric pain that has progressively become severe. Her abdomen is distended, rigid and diffusely tender. This diagnostic test should be done first:A. upright chest x-rayB. esophagogastroduodenoscopyC. abdominal ultrasound D. abdominal CT scan

(Classification - Application; Source – Schwartz’s Textbook of Surgery (8th edition): pp. 959-960) 19. After aggressive fluid therapy, a trauma victim shows a systolic BP of 110

mm Hg, cold extremities, rapid but strong peripheral pulse, and a central venous pressure of 12 cmH2O. The patient has:A. increased systemic vascular resistanceB. decreased stroke volume indexC. decreased cardiac indexD. excessive cardiac preload

(Classification - Application; Source – Schwartz’s Textbook of Surgery (8th edition): p. 132)

20. An elderly man admitted with a diagnosis of sigmoid volvulus has a markedly distended, non-tender abdomen and hyperactive bowel sounds. The initial treatment is:A. endoscopic detorsionB. detorsion via laparotomy C. decompressing transverse loop colostomyD. Re section of involved segment of colon

(Classification - Recall; Source – Schwartz’s Textbook of Surgery (8th edition): p. 1098)

21. Pancreatic necrosis is suspected in a 49-year-old woman who has not improved despite 3 days of intensive care for acute pancreatitis. The diagnosis is best established through:A. C-reactive protein measurementB. abdominal ultrasoundC. contrast-enhanced CT scanD. CT-guided percutaneous biopsy

(Classification - Recall; Source – Schwartz’s Textbook of Surgery (8th edition): pp. 1234-1238)

22. A chronically constipated 67-year-old woman presents with acute left lower quadrant (LLQ) abdominal pain, LLQ direct and rebound tenderness, and fever. The appropriate diagnostic examination is:A. transvaginal ultrasoundB. abdominal CT scanC. proctosigmoidoscopyD. barium enema

(Classification - Application; Source – Schwartz’s Textbook of Surgery (8th edition): pp. 1082-1083)

23. After an elective hemicolectomy for colon cancer, a 78-year-old man remains hypotensive and tachycardic. He has distended neck veins, cold skin, oliguria, and elevated central venous pressure. He apparently is suffering from this type of shock:A. hypovolemicB. cardiogenicC. septic

4

Page 5: Surgery- board exam questions

D. neurogenic(Classification - Application; Source – Schwartz’s Textbook of Surgery (8th edition): pp. 95-100)

24. For the past 6 weeks, a 67-year-old man has been asymptomatic except for constipation after a course of antibiotic therapy for left lower quadrant abdominal pain. He should undergo:A. CEA determinationB. barium enemaC. colonoscopyD. abdominal CT scan

(Classification - Application; Source – Schwartz’s Textbook of Surgery (8th edition): pp. 1082-1083)

25. A 32-year-old-man with a week’s history of fever presently has right lower quadrant abdominal pain and tenderness and bloody diarrhea. The most probable diagnosis is:A. typhoid ileitisB. amebic colitisC. ileocecal TBD. intussusception

(Classification - Application; Source – Schwartz’s Textbook of Surgery (8th edition): pp. 1049-1050)

26. A 75 year old woman in the ICU after undergoing cholecystectomy for acute Cholecystitis is hypotensive and tachycardic. Pulmonary capillary wedge pressure (PCWP) is elevated to 18 mmHg, and cardiac output is 3 L/min. she is shock best described as which of the following?A. hypovolemic shockB. septic shockC. cardiogenic shockD. anaphylactic shock

(Classification -Recall; Source – Schwartz’s Textbook of Surgery (8th edition): pp.)

27. A 19 year old man is brought to the emergency department with a stab wound at the base of the neck (zone I) The most important concern for patients with such injuries is which of the following?A. upper extremity ischemiaB. cerebral infarctionC. exsanguinating hemorrhageD. mediastinitis

(Classification -Recall; Source – Schwartz’s Textbook of Surgery (8th edition): pp.)

28. In septic shock, which of the following is true?A. the mortality rate is beteeen 10& and 20%B. gram-negative organisms are involved exclusivelyC. the majority of patients are elderlyD. the most common source of infection is the alimentary tract.

(Classification - Recall; Source – Schwartz’s Textbook of Surgery (8th edition): pp.)

29. The injury most often missed by selective nonoperative management of abdominal stabwounds is to which of the following?A. colonB. spleen

5

Page 6: Surgery- board exam questions

C. ureterD. diaphragm

(Classification - Recall; Source – Schwartz’s Textbook of Surgery (8th edition): pp. )

30. A 63-year-old male tobacco smoker has a 1.5 cm non-healing ulcer in his lower lip. A nodular lesion is palpable deep to the ulcer. The most likely diagnosis is:A. squamous cell carcinomaB. keratoacanthomaC. malignant fibrous histiocytomaD. verrucous carcinoma

(Classification - Application; Source – Schwartz’s Textbook of Surgery (8th edition): p. 518)

31. A 43-year-old-man presents with nasal obstruction and occasional epistaxis. Imaging studies combined with endoscopic biopsy have led to the diagnosis of nasopharyngeal cancer. The standard treatment is:A. intracavitary radiationB. external beam radiationC. combined chemotherapy and radiationD. surgical resection

(Classification - Recall; Source – Schwartz’s Textbook of Surgery (8th edition): pp. 1049-1050)

32. Biopsy of a slow growing infraauricular mass reveals a benign pleomorphic adenoma that is confined to the superficial lobe of the parotid gland. The standard treatment is:A. tumor enucleationB. tumor excision with 2 mm marginC. superficial parotidectomyD. total parotidectomy

(Classification - Application; Source – Schwartz’s Textbook of Surgery (8th edition): pp. 538-540)

33. A 58 year old woman undergoes excision biopsy of a tumor in the left posterior triangle of her neck. Histology suggests that this is a metastatic cancer. What is the most likely site of the primary tumor?A. ovaryB. adrenal glandC. kidneyD. piriform fossa

(Classification - Recall; Source – Schwartz’s Textbook of Surgery (8th edition): pp.)

34. A 59 year old woman has discomfort in the posterior part of her tongue. A biopsy confirms that the lesion is a carcinoma. What is true in carcinoma of the posterior third of the tongue?A. lymphoid tissue is absentB. lymph gland spread is often encounteredC. there is an excellent prognosisD. the tissue is well differentiated

(Classification - Recall; Source – Schwartz’s Textbook of Surgery (8th edition): pp.)

6

Page 7: Surgery- board exam questions

35. A 60-year-old man has a 3 cm nodular lesion with central ulceration in his left cheek. Section biopsy proves this to be basal cell carcinoma. This is best managed with:A. external beam radiationB. topical 5-fluouracilC. electrodessicationD. excision with 2-4 mm margin

(Classification - Application; Source – Schwartz’s Textbook of Surgery (8th edition): pp. 439-440)

36. This is the most common aggressive primary malignant bone tumor in adolescent and occurs in methaphyseal area of long bones with high incidence of pulmonary metastases:a. Chrodrosarcomab. Osteosarcomac. Fibrosarcomad. Adamantinoma

(Classification -Recall; Source – Schwartz’s Textbook of Surgery (8th edition): pp.)

37. A 65 year-old farmer presents with a 1.5-cm ulcerated lesion on the middle third of his lower lip. The lesion has been present for 4 months and is not painful. No lymph nodes are palpable in the patient’s neck. The most likely diagnosis is:

a. Squamous cell carcinomab. Basal cell carcinomac. Herpes simplexD. Keratoacanthoma

(Classification – Application; Source - PRINCIPLES OF SURGERY by SCHWARTZ, SPENCER, SHIRES 7TH EDITION, Vol. 1, 521-522)

38. A 65 year-old patient who spends summer in Baguio City presents with a painless, ulcerated lesion on the right cheek. The lesion has been present for one year. Physical examination of the patient’s neck reveals no lymph node enlargement. The most likely diagnosis is:

a. pyogenic granulomab. melanomac. basal cell carcinomaD. squamous cell carcinoma

(Classification – Application; Source - PRINCIPLES OF SURGERY by SCHWARTZ, SPENCER, SHIRES 7TH EDITION, Vol. 1, p.522)

39. A 60-year-old woman has a 10-day history of cough and fever. Imaging studies show multiple fluid loculations in the right chest cavity with an estimated volume of 500 ml. Thoracentesis draws purulent material. The best treatment strategy now is:A. pigtail catheter drainage of the empyema B. video assisted thoracoscopic drainage with deloculationC. insertion of multiple thoracostomy tubes D. early thoracotomy and drainage

(Classification - Application; Source – Schwartz’s Textbook of Surgery (8th edition): pp. 601-602)

40. A-50-year-old woman presents with a 3 cm solitary pulmonary nodule in the left upper lobe. She has a history of total thyroidectomy for papillary cancer at the age of 38 years. The main consideration is:A. TB granuloma

7

Page 8: Surgery- board exam questions

B. hamartomaC. primary lung cancerD. metastatic thyroid cancer

(Classification - Application; Source – Schwartz’s Textbook of Surgery (8th edition): pp. 556-557)

41. A 45-year-old woman presents with weakness towards the afternoon, shortness of breath, and ptosis. CT scan reveals an anterior mediastinal mass. The most likely diagnosis is:A. intrathoracic goiterB. non-Hodgkin’s lymphomaC. non-seminomatous germ cell tumorD. thymoma

(Classification - Application; Source – Schwartz’s Textbook of Surgery (8th edition): pp. 593-594)

42. A man with malignant pleural effusion in association with an inoperable lung cancer may benefit from the instillation of sclerosing agent into the pleural cavity, once the lungs are almost fully expanded. An accepted sclerosant is:

A. talcB. gentamicinC. penicillin GD. Vitamin C

(Classification - Application; Source – Schwartz’s Textbook of Surgery (8th edition): pp. 600-601)

43. A 20-year-old woman consults for a 2-cm mass in her left breast. The mass is movable, non-tender and has a rubbery consistency and smooth borders. The probable diagnosis is:A. fibroadenomaB. fibrocystic diseaseC. carcinomaD. cystosarcoma phyllodes

(Classification - Application; Source – Schwartz’s Textbook of Surgery (8th edition): pp. 463-464)

44. A 16-year-old girl is bothered by cyclical premenstrual pain in her breasts. She has 0.5-1.0 cm nodularities in the upper outer quadrant of both breasts. What management advice should be given to her?A. ObservationB. breast ultrasoundC. mammographyD. aspiration biopsy

(Classification - Application; Source – Schwartz’s Textbook of Surgery (8th edition): pp. 465-466)

45. An asymptomatic 40-year-old woman with no palpable breast mass undergoes mammography. This reveals clustered microcalcifications in the lower outer quadrant of her left breast. The probable diagnosis is:A. florid hyperplasiaB. sclerosing adenosisC. atypical ductal hyperplasiaD. ductal carcinoma in situ

(Classification - Application; Source – Schwartz’s Textbook of Surgery (8th edition): pp. 472-473)

8

Page 9: Surgery- board exam questions

46. A 45-year-old woman with “lumpy” breasts undergoes mammography. Mammographic findings are suggestive of high-grade ductal carcinoma in situ confined to the upper outer quadrant of her right breast. How should she be managed?A. lumpectomy aloneB. lumpectomy and radiotherapyC. mastectomyD. mastectomy and axillary node sampling

(Classification - Application; Source – Schwartz’s Textbook of Surgery (8th edition): p. 481)

47. A 45-year-old woman has a recent onset bloody nipple discharge from her right breast. No palpable breast mass is noted. What diagnostic test is indicated?A. breast ultrasoundB. mammographyC. ductographyD. cytology of discharge

(Classification - Application; Source – Schwartz’s Textbook of Surgery (8th edition): p. 493)

48. A premenopausal woman undergoes modified radical mastectomy for a 3-cm breast cancer. No axillary node and distant metastases are detected. Test for this biomarker is currently recommended to facilitate the selection of adjuvant chemotherapy:A. c-fosB. c-mycC. p53D.AHER2/neu

(Classification - Application; Source – Schwartz’s Textbook of Surgery (8th edition): p. 493)

49. A 63-year-old man with chronic atrial fibrillation has sudden onset of pain, weakness, and paresthesia in his left leg, which appears cool, cyanotic, and without femoral and distal pulses. The right leg has normal pulses. The most likely diagnosis is:A. arterial embolismB. aortoiliac thrombosisC. Buerger’s diseaseD. Raynaud’s disease

(Classification - Application; Source – Schwartz’s Textbook of Surgery (8th edition): pp. 759-762) 50. Coronary angiogram reveals a triple-vessel disease in a diabetic 55-year-

old man presenting with unstable angina. The recommended treatment is:A. catheter-directed thrombolysisB. balloon angioplastyC. coronary artery bypass graftingD. Transmyocardial revascularization

(Class;ification - Application; Source – Schwartz’s Textbook of Surgery (8th edition): pp. 652-653) 51. A 41-year-old male chronic smoker has a 2-year history of bilateral foot

claudication. He now has ulceration of the tip of the right 3rd toe and left 2nd

and 4th toes. Popliteal pulses are palpable but the posterior tibial and dorsalis pedis pulses are bilaterally absent. The most important step in management is:

9

Page 10: Surgery- board exam questions

A. cessation of smokingB. long-term anticoagulant therapyC. multiple toe amputationsD. angiography followed by bypass surgery

(Classification - Application; Source – Schwartz’s Textbook of Surgery (8th edition): pp. 792-793)

52. A premature infant with progressive dyspnea without cyanosis since birth has machinery murmur in the pulmonic area. A diagnosis of patent ductus arteriosus (PDA) is made. The indicated treatment is:A. administration of indomethacinB. infusion of prostaglandinC. administration of indomethacinD. immediate surgical correction of PDA

(Classification - Recall; Source – Schwartz’s Textbook of Surgery (8th edition): pp. 617-618)

53. A 3-year-old girl presents with cyanotic spells that have increased in severity and frequency since birth. X-ray shows a boot-shaped heart; ECG reveals right ventricular hypertrophy. The most probable diagnosis is:A. truncus arteriosusB. tetralogy of FallotC. transposition of the great arteriesD. Ebstein’s anomaly

(Classification - Application; Source – Schwartz’s Textbook of Surgery (8th edition): p. 636)

54. A 54-year-old man presents with hematemesis after a bout of vomiting and retching. Endoscopy shows linear mucosal tears at the gastroesophageal junction. The diagnosis is:A. Mallory-Weiss syndromeB. Boerhaave’s syndrome C. Menetrier’s diseaseD. Dieulafoy’s lesion

(Classification - Recall; Source – Schwartz’s Textbook of Surgery (8th edition): p. 919)

55. After truncal vagotomy and antrectomy with gastrojejunostomy for an obstructed duodenal ulcer, the patient complains of diaphoresis, weakness, and abdominal discomfort followed by diarrhea several minutes after meals. The symptoms are suggestive of:A. post-vagotomy diarrheaB. bile reflux gastritisC. afferent loop syndromeD. dumping syndrome

(Classification - Recall; Source – Schwartz’s Textbook of Surgery (8th edition): pp. 985-987)

56. A woman undergoes total gastrectomy for a huge proximal gastric carcinoma. To protect her from developing anemia, she must be given a regular parenteral dose of:A. folic acidB. ferrous sulfateC. vitamin B12D. transferrin

(Classification - Recall; Source – Schwartz’s Textbook of Surgery (8th edition): p. 990)

10

Page 11: Surgery- board exam questions

57. A few weeks after recovering from severe pancreatitis, a 34-year-old man has developed a pancreatic pseudocyst with mature wall that is pressing on his stomach. Aspiration fails to relieve the pressure symptoms. The indicated treatment is:A. administration of somatostatinB. internal drainage (cystogastrostomy) C. external drainageD. excision of pseudocyst

(Classification - Application; Source – Schwartz’s Textbook of Surgery (8th edition): pp. 1256-1258)

58. ERCP reveals a periampullary cancer in a 64-year-old man admitted for jaundice. No metastasis is detected; no co-morbid conditions are identified. How should he b managed?A. radical excision of the head of pancreas and duodenumB. local excision and adjuvant chemotherapyC. external beam radiationD. stenting and chemotherapy

(Classification - Application; Source – Schwartz’s Textbook of Surgery (8th edition): pp. 1281-1288)

59. A 60 year old man with no previous operation has a 5-day history of inability to pass flatus or feces, cramping abdominal pain, and progressive abdominal distention. The principal diagnostic consideration is:A. midgut volvulusB. sigmoid volvulusC. colorectal cancerD. ileocecal tuberculosis

(Classification - Application; Source – Schwartz’s Textbook of Surgery (8th edition): pp.)

1. A 65 year old male who underwent a colon resection of carcinoma 2 years prior to consult is found to have 3 solid nodules approximately 2 cms each in the right and left lobes of the liver. There is no extrahepatic disease detected. The preferred treatment is:A. systemic chemotherapyB. radiofrequency ablationC. microwave coagulation therapyD. surgical resection of metastatic nodules

(Classification – Application; Source – Schwartz’s Textbook of Surgery (8th edition): pp.)

2. A 35 year old female with a prolonged intake of contraceptive pills develops right upper quadrant pains. Ultrasound shows an isodense 5 cm mass in the right lobe of the liver. Sulfur-colloid scan done showed a cold lesion. This patient should undergo.A. conservative treatmentB. resectionC. enucleationD. liver transplantation

(Classification - Application; Source – Schwartz’s Textbook of Surgery (8th edition): pp. 119-120)

60. A man who has an obstructed rectosigmoid cancer in association with a competent ileocecal valve is liable to develop perforation of:A. sigmoid colon

11

Page 12: Surgery- board exam questions

B. splenic flexureC. hepatic flexureD. cecum

(Classification - Application; Source – Schwartz’s Textbook of Surgery (8th edition): pp. 1089-1090)

61. The presence of tenesmus, decrease in caliber of stools and occasional bloody-mucoid diarrrrhea in a 67-year-old man is suggestive of:A. rectal carcinomaB. amebic colitisC. intussusceptionD. ileocecal tuberculosis

(Classification - Application; Source – Schwartz’s Textbook of Surgery (8th edition): pp. 1090-1091)

62. Diagnostic tests show an ileocolic intussusception in a 38-year-old man, who has no signs of strangulation. What is recommended for this patient?A. expectant treatmentB. endoscopic decompressionC. hydrostatic reductionD. exploratory laparotomy

(Classification - Application; Source – Schwartz’s Textbook of Surgery (8th edition): pp.)

63. A 63-year-old man is presently asymptomatic after medical treatment for LLQ pain and tenderness. He should undergo:A. CEA determinationB. ColonoscopyC. barium enemaD. abdominal CT scan

(Classification - Application; Source – Schwartz’s Textbook of Surgery (8th edition): pp.)

64. A 56-year-old woman taking NSAID for chronic arthritis experienced severe epigastric pain 6 hours ago. Her abdomen is now diffusely tender with board-like rigidity. The appropriate initial diagnostic test is:A. chest upright x-rayB. plain abdomen supine x-rayC. abdominal ultrasoundD. abdominal CT scan

(Classification - Application; Source – Schwartz’s Textbook of Surgery (8th edition): pp.)

65. Despite 3 months of medical treatment for posterior midline anal fissure, the man’s fissure-related anal pain during and after defecation has become excruciating. The appropriate treatment is:A. excision of fissureB. botulinum toxin injectionC. anal divulsionD. internal sphincterotomy

(Classification - Application; Source – Schwartz’s Textbook of Surgery (8th edition): pp. 1103-1104)

66. During surgery for a large right indirect inguinal hernia, the cecum is noted to form part of the wall of the hernia sac. The patient has this type of hernia:A. sliding

12

Page 13: Surgery- board exam questions

B. Richter’sC. interstitialD. Spigelian

(Classification - Application; Source – Schwartz’s Textbook of Surgery (8th edition): p. 1358)

67. A previously asymptomatic 45-year-old obese woman consults for a non-tender swelling below her right inguinal ligament just lateral to the pubic tubercle. A warranted diagnostic procedure for this patient is:A. fine needle aspirationB. ultrasoundC. MRID. herniography

(Classification - Application; Source – Schwartz’s Textbook of Surgery (8th edition): pp. 1364-1366)

68. A 54 year old male taking anticoagulant for the fast 4 weeks developed 3 day history of abdominal pain. On physical examination there was a palpable mass at the supraumbilical area with direct tenderness. Fothergill sign way positive. What is your impression?A. desmoid tumorB. rectus sheath HematomaC. omental torsionD. omental infarction

(Classification - Application; Source – Schwartz’s Textbook of Surgery (8th edition): pp.)

69. A 12 year old female complain of 1 day history of epigastric pain which shifted to the right lower quadrant associated with anorexia and vomiting. Physical examination revealed direct and rebound tenderness on the right lower quadrant. With the preoperative diagnosis of acute appendicitis, she was operated on. However, intraoperatively there was blood tinged peritoneal fluid and appendix was noted to be normal. The most probable cause of the patient condition is:A. mesenteric cystB. omental torsionC. ruptured rectus sheath HematomaD. mesenteric adenitis

(Classification - Application; Source – Schwartz’s Textbook of Surgery (8th edition): pp.)

70. A 45-year-old is discovered to have a solitary 2-cm firm nodule in his right thyroid lobe. No associated symptoms are noted; family history is negative for thyroid cancer. Diagnostic work-up should start with:A. thyroid function testsB. thyroid ultrasoundC. thyroid scanD. fine needle aspiration biopsy

(Classification - Application; Source – Schwartz’s Textbook of Surgery (8th edition): pp. 1413-1416)

71. A 59-year-old man presents with 4-cm firm mass in the left lobe of the thyroid associated with a firm occipital mass that is 6 cm in its widest diameter, which he has neglected for the past 9 months. Family history is negative for thyroid malignancy. He probably has this type of thyroid cancer:A. papillary

13

Page 14: Surgery- board exam questions

B. follicularC. medullaryD. anaplastic

(Classification - Application; Source – Schwartz’s Textbook of Surgery (8th edition): pp. 1420-1421)

72. A 49-year-old woman complains of central obesity with moonlike facies and hirsutism. Her ACTH is markedly elevated, which is suppressed by large dose of dexamethasone. The probable pathology is:A. adrenal adenomaB. adrenal carcinomaC. pheochromocytomaD. pituitary adenoma

(Classification - Application; Source – Schwartz’s Textbook of Surgery (8th edition): pp. 1455-1458)

73. A 42-year-old man presents with headache, palpitations, and diaphoresis. His urinary metanephrines are elevated. Imaging studies reveal a 3-cm mass in his right adrenal gland. The treatment of choice is:A. adrenalectomyB. radiotherapyC. tumor embolizationD. chemoradiation

(Classification - Application; Source – Schwartz’s Textbook of Surgery (8th edition): pp. 1460-1461)

74. A 67-year-old man is bleeding excessively following a transurethral prostatectomy. The bleeding is assessed to be due to local plasminogen activation leading to increased fibrinolysis on the raw wound surface. Administration of this drug may be beneficial:A. vitamin KB. heparinC. warfarinD. EACA

(Classification - Recall; Source – Schwartz’s Textbook of Surgery (8th edition): p. 82)

75. A 60-year-old man presents with slight urinary urgency, frequency, and a decrease in the force of micturition. Digital rectal examination detects no mass in the prostate. The indicated diagnostic test is:A. prostatic acid phosphataseB. prostate-specific antigenC. pelvic ultrasoundD. needle biopsy of prostate

(Classification - Application; Source – Schwartz’s Textbook of Surgery (8th edition): pp. 1535-1537)

76. A 35-year-old man presents with right flank pain and microscopic hematuria. Intravenous pyelography reveals a 4 mm radiolucent stone in the distal right ureter. The recommended treatment is:A. alkalinization of the urineB. extracorporeal shock wave lithotripsyC. stone extraction via ureteroscopeD. open ureterolithotomy

(Classification - Application; Source – Schwartz’s Textbook of Surgery (8th edition): pp. 1546-1549)

14

Page 15: Surgery- board exam questions

77. A 55-year-old man is admitted for gross hematuria associated with left flank pain and anemia. CT scan detects a solid, enhancing mass in the left kidney. The likely diagnosis is:A. renal cell carcinomaB. renal tuberculosisC. renal sarcomaD. renal oncocytoma

(Classification - Application; Source – Schwartz’s Textbook of Surgery (8th edition): pp. 1531-1532)

78. A 23-year-old woman sustained blunt head trauma to her right frontoparietal area in a car crash, which caused he to be unconscious for several minutes. Upon arrival at the E.R., she is alert and has no neurologic deficit. X-ray shows a linear non-depressed right frontoparietal fracture. An hour after admission, vomiting followed by progressive neurologic deterioration are noted.A. diffuse axonal injuryB. cerebral contusionC. acute epidural hematomaD. acute subdural hematoma

(Classification - Application; Source – Schwartz’s Textbook of Surgery (8th edition): p. 1619)

79. A 54-year-old woman with no previous neurologic disorder has new onset seizures without associated gross neurologic deficit. She was treated for breast cancer 10 years ago. If brain metastasis is the main consideration, this is best confirmed by means of:A. electroencephalographyB. CT scan of the brain with intravenous contrastC. MRI of the brain with intravenous contrastD. Doppler ultrasound

(Classification - Recall; Source – Schwartz’s Textbook of Surgery (8th edition): p. 1633)

80. A 55-year-old man with a history of lung cancer presents with frequent severe headache, occasional vomiting, and weakness of the right upper extremity. What is the recommended treatment in addition to corticosteroids?A. intrathecal chemotherapyB. whole-brain radiotherapyC. stereotactic radiosurgery (gamma knife)D. resection of lesion causing arm weakness

(Classification - Application; Source – Schwartz’s Textbook of Surgery (8th edition): p. 1633)

81. A young man just admitted for blunt injuries from a car accident is stuporous. CT scan shows cerebral contusion. What should be avoided as it can cause secondary brain injury to this patient through its deleterious effect on intracranial pressure? A. elevation of the headB. respiratory acidosisC. hypovolemia even of mild degreeD. sedation of agitated patient

(Classification - Recall; Source – Schwartz’s Textbook of Surgery (8th edition): pp. 1613-1614)

15

Page 16: Surgery- board exam questions

82. As a basketball player falls hard on his extended right arm, he feels immediate severe pain with subsequent inability to move his right arm. Swelling and tenderness about the shoulder with loss of deltoid contour are noted. The likely diagnosis is:A. deltoid muscle ruptureB. shoulder dislocationC. fracture of proximal humerusD. brachial plexus injury

(Classification - Application; Source – Schwartz’s Textbook of Surgery (8th edition): pp. 1706-1707)

83. A 10-week-old baby with cleft lip and palate has undergone a successful cleft lip repair. It is recommended that cleft pate repair be done before the baby reaches this age:A. 6 monthsB. 12 monthsC. 2 yearsD. 4 years

(Classification - Recall; Source – Schwartz’s Textbook of Surgery (8th edition): pp. 1797-1799)

84. A bedridden 61-year-old woman has a significant neurologic recovery from a previous stroke. She has an infected deep pressure ulcer in her sacrum. Once the infection has been controlled by debridement and supportive treatment, the pressure ulcer can be managed with:

A. skin graftB. rotation skin flapC. gluteus maximus flapD. microsurgery free flap

(Classification - Recall; Source – Schwartz’s Textbook of Surgery (8th edition): pp. 1825-1826))

85. A small thick antecubital scar from a 2nd degree flame burn is constricting movement. To relieve the constriction, the appropriate procedure to refashion the scar is:A. Z-plastyB. advancement flapC. scar excision and skin graftD. deltopectoral flap

(Classification - Recall; Source – Schwartz’s Textbook of Surgery (8th edition): pp. 1792-1793))

86. Resection of a sarcoma results in a significant lateral chest wall defect. To minimize pulmonary dysfunction, chest wall reconstruction may utilize:A. rectus abdominis muscle flapB. omental flapC. tensor fascia lata graftD. Marlex mesh

(Classification - Recall; Source – Schwartz’s Textbook of Surgery (8th edition): pp. 1823-1824))

87. A 4-week-old baby boy presents with frequent non-bilious vomiting. During feeding, peristaltic waves are seen in the upper abdomen in association with a palpable mass in the right upper quadrant. The diagnosis is:A. hypertrophic pyloric stenosisB. duodenal atresiaC. jejunal atresia

16

Page 17: Surgery- board exam questions

D. volvulus neonatorum(Classification - Application; Source – Schwartz’s Textbook of Surgery (8th edition): pp. 1486-1487)

88. A newborn infant has excessive drooling followed by coughing immediately after feeding. The main diagnostic consideration is esophageal atresia. This can be readily confirmed by:A. inability to pass orogastric tube into the stomachB. water-soluble contrast study (esophagram)C. transesophageal ultrasoundD. plain abdominal radiograph

(Classification - Application; Source – Schwartz’s Textbook of Surgery (8th edition): pp. 1481-1482))

89. A diagnosis of intussusception is made in an infant with paroxysms of crampy abdominal pain and vomiting. The infant has no peritoneal signs and remains hemodynamically stable. The initial treatment of choice is:A. nasogastric decompressionB. endoscopic reductionC. pneumatic reductionD. hydrostatic reduction with barium

(Classification - Recall; Source – Schwartz’s Textbook of Surgery (8th edition): pp. 1493-1494)

90. A 3-day old infant who has not passed meconium presents with abdominal distention and bilious vomiting. The most likely diagnosis is:A. duodenal atresiaB. malrotationC. midgut volvulusD. Hirschprung’s disease

(Classification - Application; Source – Schwartz’s Textbook of Surgery (8th edition): pp. 1496-1497)

91. A 3-week-old infant is noted to have jaundice in association with acholic stools shortly after birth. A radionuclide scan demonstrates presence of radioisotope in the intestine. This finding rules out the diagnosis of:A. neonatal hepatitisB. biliary atresiaC. inspissated bile syndromeD. choledochal cyst

(Classification - Application; Source – Schwartz’s Textbook of Surgery (8th edition): pp. 1499-1500)

92. A 67 y/o man has an intraabdominal abscess caused by perforated sigmoid diverticulitis. The most common isolated microorganism is__?A. Escherichia coli B. Bacteroides C. K. pneumoniaeD. Staphylococcus epidremides

Schwartz’s: surgical infection pp 121

93. One of the most common complication of central venous access is_____.A. hemorrhageB. pneumothoraxC. wound infectionD. air embolus

Schwartz’s Complication in Surgery p.337

17

Page 18: Surgery- board exam questions

94. Early postoperative small bowel obstruction is a rare finding and occurs less than 1 % of the time. When it does, the most common cause is_____.

A. internal herniasB. post-operative adhesionsC. infections or abscessD. technical errors

Schwartz’s: surgical complications p. 347

95. Which of the ff statements is true concerning corrosive injury to the esophagus?

A. Acid ingestion is not injurious to the stomach because of its non-acidic pH.

B. Ingested caustic agents rapidly pass through the esophagus and stomach into the small intestines.

C. Alkaline injury is more destructive than acid injury.D. Acid injury is more destructive than alkaline injury.

Schwartz’s: Esophagus p. 909

96. A 27 year old female was brought to the ER with history of suicidal ingestion of household muriatic acid. This patient is best managed by---A. Give half strength vinegar, lemon juice or orange juiceB. Start with milk, egg white or antacidsC. Give emetics to induce vomitingD. Sodium bicarbonate is one of the treatment option.

Schwartz’s: Esophagus p. 910

97. Which of the following statements is true regarding the arterial blood supply of the stomach?A. The right gastric artery, a branch of the superior mesenteric artery

supplies the gastric antrumB. Because of the rich intramural collateral vessels, gastric viability

may be preserved after ligation of at least two of the four named gastric arteries

C. The largest artery supplying the stomach is the right gastric artery.D. The left gastroepiploic artery is a branch of the celiac trunk.

Schwartz’s: Stomach pp. 935-937

98. At a cellular level, the major stimulants of acid secretion by the gastric parietal cell are the following, except:

A. histamineB. prostaglandinE2C. acetylcholineD. gastrin

Schwartz’s: Stomach p. 943

99. A patient with gastric adenocarcinoma underwent subtotal gastrectomy. Histopath revealed tumor penetrating the serosa, regional lymph nodes are not involved and distant metastatic lesions are not detected. What is the correct tumor stage on TNM staging? BA. Stage IB. Stage IIC. Stage III-AD. Stage III-B

Schwartz’s: Stomach p. 970

18

Page 19: Surgery- board exam questions

100. A 5-month old baby boy was seen at the ER with complaints of non-bilous vomiting that became increasingly projectile over several days to weeks and cannot tolerate liquid intake. Abdominal examination revealed palpation of typical “olives” in the right upper quadrant with visible gastric waves. What is the correct diagnosis?A. Meconium ileus C. Hypertrophic pyloric stenosisHirschprung’s disease D. Intussusception

Schwartz’s: Pediatric Surgery p.1486

101. Which of the following is the most common malignant liver tumor in childhood?A. Hemangioma C. Hepatocellular carcinomaB. Hepatoblastoma D. Mesenchymal hamartoma

Schwartz’s: Pediatric Surgery p.1512

102. What is the appropriate treatment for an incidental 3 cm liver hemangioma found on abdominal CT scan of 37 year old woman?A. No treatment C. Selective embolization of left hepatic arteryB. Cryoablation D. Left hepatic lobectomy

Schwartz’s: pp. 1162 – 1163

103. A 35 y/o male presents with 1 year history of early satiety and right upper abdominal discomfort. CT scan shows a 6 x 8 cm cystic lesion in the right lobe of the liver. Ultrasound of cyst show no internal echoes. The appropriate management is:A. cryoablation of cyst C. unroofing of the cystB. right hepatic lobectomy D. percutaneous aspiration of cyst

Schwartz’s: p. 1159

104. An imaginary line that divides the liver into right and left lobe that runs from the inferior vena cava to the tip of the gallbladder fossa: CA. Catlie lineB. Calot’s lineC. Cantlie’s lineD. Charcot’s line

Schwartz’s: p. 1140

105. The diagnostic tool important in evaluating liver injury in a stable blunt trauma patient is:A. Ultrasound C. CT scanB. Abdominal x-ray D. Peritoneal lavage

Schwartz’s: p. 142

106. The local factor affecting wound healing:A. Age C. Low oxygen tensionB. Smoking D. Trauma

Schwartz’s: p. 235

107. The characteristic of Keloid scars:A. Arise above the skin level and stay in the confine of the original woundB. Develop within 4 weeks after traumaC. Extend beyond the border of original wound and rarely regress

spontaneouslyD. Occur in areas of flexion and extension

Schwartz’s: p. 240

19

Page 20: Surgery- board exam questions

108. A 55 y/o male diagnosed to have chronic pancreatitis, developed epigastric mass measuring about 6 cm accompanied by pain and fullness. The diagnosis is:A. Gastric carcinoma C. Pseudocyt of the pancreasB. Pancreatic carcinoma D. Acute pancreatitis

Schwartz’s: p. 1256

109. Type of benign solitary pancreatic neoplasm consisting of symptomatic fasting hypoglycemia and profound syncopal episodez:A. Insulinoma C. GastrinomaB. Glucagoma D. VIPoma

Schwartz’s: p. 1275

110. The most common indication for splenectomy:A. Staging of Hodgkin’s disease C. Trauma to spleenB. Hereditary spherocytosis D. Thalassemia

Schwartz’s: p. 1301

111. The best time to repair of cleft lip is:A. 10 months C. 6 monthsB. 10 weeks D. 16 weeks

Schwartz’s: p 1797

112. A 36 y/o female complain of an acute pain at the lateral aspect of her left breast. A tender firm cord is found on the same site on physical examination. It was diagnosed as a Mondors disease. This lesion is best managed with:A. Radiotherapy B. Anti-inflammatory medication, warm compress and rest of

ipsilateral extremityC. Immediate excision of the lesion is needed D. Total mastectomy

Schwartz’s: p. 463

113. Breast lesion that can be treated with close observation with or without tamoxifen:early invasive breast carcinoma lobular carcinoma in situ ductal carcinoma in situinflammatory carcinoma of the breast

Schwartz’s: p.481

114. The best screening imaging technique for breast cancer is: DA. Magnetic resonance imaging B. UltrasonographyC. DuctographyD. Mammography

Schwartz’s: pp. 476-477

115. A 46 y/o female who underwent a total thyroidectomy for stage II papillary carcinoma developed 3 weeks later with Chvostek’s and Trouseau’s sign. This could be due to:A. Recurrence of papillary thyroid carcinoma B. Injury to cervical sympathetic trunkC. Hypoparathyroidism D. Injury to recurrent laryngeal nerve

Schwartz’s: pp. 1429-1448

20

Page 21: Surgery- board exam questions

116. This is considered as a single most important test in the evaluation of patients with thyroid masses:A. Thyroid ultrasound B. CT and MRIC. Fine needle aspiration cytologyD. Thyroid scanning

Schwartz’s: p. 1415

117. Lymph node in the posterior triangle of the neck is within what level:A. Level IIB. Level IVC. Level VD. Level VI

Schwartz’s: p. 534

118. This form of shock has a low blood pressure, low urine output but has an elevated central venous pressure:A.Vasodilatory shockB. Obstructive shock C. Neurogenic shockD. Traumatic shock

Schwartz’s: p. 95-102

119. 55 y/o male who has been in the ward for the past 4 days and is receiving IV fluid of D5W is complaining on his 5th hospital day of headache, nausea, vomiting, anorexia, body malaie and watery diarrhea. Give the most probable electrolyte imbalance the patient might have:A. HypernatremiaB. HyperkalemiaC. HyponatremiaD. Hypokalemia

Schwartz’s: p. 48

120. The most important part of the treatment of severe metabolic acidosis among trauma patient is:A .Correct abnormality with exogenous bicarbonateB. High oxygenation E. Restore perfusion with volume resuscitation F. D. Give colloid solution

Schwartz’s: p. 50-51

121. The most common primary bone malignancy: A. OsteosarcomaB. Ewing’s sarcomaC. OstoblastomaD. Chondrosarcoma

Schwartz’s: p. 1665

122. The most potent stimulant for aldosterone release is: A. ACTHB. renninC. angiotensin ID. angiotensin II

Schwartz’s: p. 9

123. The most biologically active and potent vasoconstrictor is

21

Page 22: Surgery- board exam questions

angiotensin IIendothelinsepinephrineserotonin

Schwartz’s: p. 23

124. In trauma patients significant reduction in infectious complications were noted in patients given early enteral nutrition as compared to those given who are unfed or given parenteral nutrition except for what type of trauma patients? A. renal traumaB. long bone fracturesC. head traumaD. liver trauma

Schwartz’s: p. 31

125. Branched chain amino acids are used in parenteral nutrition for what specific type of patients?A. renal failure patientsB. diabetic patientsC. pulmonary failure patientsD. hepatic failure patients

Schwartz’s: p.33

126. In what type of parenteral feeding is fat increased to 50% of total calories?A. Heart failure formulasB. pulmonary failure formulas C. renal failure formulasD. calorie-dense formulas

Schwartz’s: p. 33

127. The appendeceal flora remains constant throughout life with the exception of what bacteria, which appears only in adults? A. Porphyromonas gingivalisB. Pseudomonas aeruginosa C. PeptostreptococciD. Streptococcus anginosus

Swhartz’s: p. 1121

128. The most significant factor associated with both fetal and maternal deaths in pregnant patients with acute appendicitis is A. formation of a phlegmonB. appendeceal perforation C. appendeceal tip near the uterusD. retrocecal appendix

Schwartz’s: p. 1129

129. Which of the ff. is true of the superior mesenteric artery syndrome? C A. It is often seen in obese individuals.B. It usually affects patients over 70 yrs. old.C. Compression is over the third portion of the duodenum.D. It is a surgical emergency.

Swhartz’s: p. 1027

130. Which of the following factors present in a entero-cutaneous fistula increases the possibility of closure of the tract?A. non-epithelialization of the tract

22

Page 23: Surgery- board exam questions

B. high output fistula C. malnutritionD. sepsis

Swhartz’s: p. 1037

131. What is the single most important factor in predicting burn related morbidity and mortality?A. size of burnB. type of burnC. etiology of burnD. associated medical condition

Schwartz 7th ed., p. 228-232 / Ans. A

132. What type of shock is caused by the interference of the balance of vasolidator and vasoconstrictor influences to arterioles and venules?A. cardiogenic shock B. hypovolemic shockC. septic shockD. neurogenic shock

Shwartz 7th ed., p. 113-115 / Ans. D

133. A 50 y/o female consulted to you and complaints of fatigue, polydypsia, polyuria, nocturia, joint pains and constipation. Her laboratory exams shows normal FBS, elevated serum calcium, and elevated intact PTH. What would be the most likely diagnosis?A. NIDDMB. Metastatic breast caC. HyperthyroidismD. Hyperparathyroidism

Shwartz 7th ed., p. 1434-1439 / Ans. D

134. A 35 y/o male presents with a long standing severe hypertension with associated muscle weaknes, headache, polyuria and polydypsia. Blood exams revealed an elevated aldesterone level and elevated serum potassium. What is the most likely diagnosis?A. cushing’s syndromeB. pheochromocytomaC. conn’s syndromeD. waterhouse-friderichsen syndrome

Shwartz 7th ed., p. 1453-1458 / Ans. C

135. Which of the following is the most commonly inured organ in blunt abdominal injury?A. small intestineB. large intestineC. pancreasD. spleen

Schwartz 7th ed., p. 187 / Ans. D

136. In the initial management of an acutely and seriously injured patient, the first and most important emergency measure to be taken care of is:A. splinting of fractureB. control of bleedingC. restoration of blood volumeD. ensuring an adequate airway

Schwatz 7th ed., p. 156 / Ans. D

23

Page 24: Surgery- board exam questions

137. Diagnostic peritoneal lavage (DPL) remains the most sensitive test available for determining the presence of intraabdominal injury. The result of DPL is considered grossly positive if;A. 1 ml of blood is aspiratedB. 5 ml of blood is aspiratedC. 8 ml of blood is aspiaratedD. 15 ml of blood is aspirated

Schwartz 7th ed., p. 167 / Ans. D

138. Which of the following constitute an immediate threat to life because of inadequate ventilationA. flail chestB. hemothoraxC. stabbed wound involving the diaphragmD. pulmonary hematoma

Schwartz 7th ed., p.156-157 / Ans. A

139. While undergoing exploratory laparotomy for blunt abdominal injury, to the patients BPwas noted to be 70 palpatory while bleeding was noted coming from the splenic hilum. Appropriate management for the injured spleen should be;A. splenectomyB. splenorrhapyC. debridement and repairD. packing, immediate closure of abdominal lesion and volume

replacementSchwartz 7th ed., p. 193 / Ans. A

140. During the initial resuscitation of a hypovolemic patient secondary to a vehicular accident external bleeding is best controlled by;A. suturingB. direct finger pressureC. application of tourniquetD. blood transfusion

Schwartz 7thed., p. 157 / Ans. B

141. In penetrating neck injuries zone II is referred to an area between;A. clavicle and cricoid cartilageB. cricoid cartilage and angle of mandibleC. above the angle of the mandibleD. below the clavicle

Schwartz 7th ed., p. 165 / Ans. B

142. In assessing the extent of hepatic injury following blunt injury to the abdomen, bleeding from the liver is best controlled by;A. manual cmpression of liver parenchymaB. using figure of 8 - sutureC. pringle maneuverD. liver resection

Schwartz 7th ed., p. 188-189 / Ans. C

143. Stable patients at risk of urethral injury, manifestating as presence of blood at the matus, necessitates an immediate;A. urethral catheterizationB. urethrographyC. cystoscopyD. KUB-IVP

24

Page 25: Surgery- board exam questions

Schwartz 7th ed., p. 161-162 / Ans. B

144. Type of healing in which the wound is allowed to heal by granulation tissues formation and contraction:A. primary intentionB. secondary intentionC. tertiary intentionD. quarternary intention

Schwartz 8th ed., p. 234 / Ans. B

145. Partial thickness wound such as seen in superficial second degree burns heal by which of the following process:A. wound contractionB. epithelizationC. granulation tissue formationD. maturation and remodeling

Schwartz 8th ed., p. 228 / Ans. B

146. Equilibrium between collagen synthesis and collagen degradation occurred during the stage of;A. hemostasis and inflammationB. proliferative stageC. matrix synthesisD. remodeling

Schwartz 8th ed., p. 228 / Ans. D

147. Delayed primary wound closure is indicated in which of the following type of wound;A. diabetic ulcerB. decubitus ulcerC. contaminated traumatic woundD. surgical incision following simple appendectomy

Schwartz 7th ed., p. 264 / Ans. C

148. Cytokines that are produced by one cell and affect an adjacent cell such as transforming growth factor beta (TGF-B) are called;A. endocrine factorsB. paracrine factorsC. autocrine factorsD. intacrine factors

Schwartz 7th ed., p. 266 / Ans. B

149. The major component of the extracellular matrix that provide strength, support and structure of all soft tissues, tendons, ligaments and bones is;A. collagenB. elastinC. fibronectinD. hyaluronic acid

Schwartz 7th ed., p. 270-271 / Ans. A

150. The genetic disorder arising from mutations in the genes for type I collagen causing increase propensity for the bones to break under minimal stress is;A. ehlers-danlos syndromeB. osteogenesis imperfectaC. marfan’s syndromeD. epidermolysis bullosa

25

Page 26: Surgery- board exam questions

Schwartz 7th ed., p. 278 / Ans. B

151. Lesion of the gastrointestinal tract that result in comlete regeneration and recovery;A. peptic ulcer diseaseB. crohns diseaseC. radiation colitisD. gastric erosions

Schwartz 7th ed., p. 280-281 / Ans. D

152. Skin lesions that extend beyond the boundaries of the original wound, do not regress with time and recur after excision;A. hypertrophic scarsB. keloidsC. diabetic ulcersD. decubitus ulcers

Schwartz 7th ed., p. 281-282 / Ans. B

153. Kehr’s sign is a classic example of;A. unreferred visceral painB. referred visceral painC. unreferred parietal painD. referred parietal pain

Scwratz 7th ed., p. 1034-1035 / Ans. D

154. The bleeding in Mallory-Weiss syndrome is secondary to;A. penetration of ulcer affecting the gastroduodenal arteryB. linear tears of the gastroesophageal junctionC. gastric erosionsD. hematobilia

Schwartz 7th ed., p. 1062-1063 / Ans. B

155. The most common cause of intestinal obstruction for all age group combined is;

A. strangulated herniaB. volvulusC. adhesive bandsD. neoplasm

26

Page 27: Surgery- board exam questions

Schwartz 7th ed., p. 1054 / ans. C156. The most obvious route of fuid and electrolyte loss in patients with intestinal obstruction

from leocecal TB is through;the edematous bowel wallthe intestinal lumenvomitingthe peritoneal cavity

Schwartz 7th ed., p . 1056 / Ans. C

157. Occlusion of the blood supply to a segment of bowel in addition to obstruction of the lumen is referred to as;

A. complete intestinal obstructionB. closed-loop obstructionC. strangulated obstructionD. ileus

Schwartz 7th ed., p. 1057 / Ans. C

158. Which of the following diagnostic modalities consistently localizes the site of bleeding in cases of lower GI bleeding;

A. colonoscopyB. CT scanC. AngiographyD. Barium enema

Schwartz 7th ed., p. 1066 / Ans. C

159. The most common cause of massive lower GI bleeding is;A. colonic malignancyB. diverticulosisC. adenamatous polypsD. ulcerative colitis

Schwartz 7th ed., p. 1065 / Ans. B

160. A 50 y/o male with history of alcoholism was admitted because of history of progressive jaundice, associated with tea-colored urine, achlic stools, weight loss and mild epigastric pain. The simplest and most non-invasive method in the diagnosis of this patient is;

A. percutaneous transhepatic cholangiography (PTC)B. ERCPC. UltrasonographyD. HIDA scan

Schwartz 7th ed., p. 1071 / Ans. C

161. Relative to the ileum, the jejunum has;A. more fatly mesenteryB. less prominent plicae circularisC. longer vasa rectaD. lesser in diameter

Schwatz 8th ed., p . 1018 / Ans. C

162. The most common malignant neoplasm of the small bowel is;adenocarcnomacarcinoidssarcomaslymphomas

Schwartz 7th ed., p. 1242 / Ans. A

163. The presence of a Meckels diverticulum in a hernial sac is called;

Page 28: Surgery- board exam questions

A. spingelian herniaB. femoral herniaC. littre’s herniaD. richter’s hernia

Schwartz th ed., p. 1249 / ans. C

164. Rovsings’ sign is elicited by;A. pain the right lower quadrant when palpatory pressure is exerted in the left

lower quadrantB. pain in the right lower quadrant when palpatory pressure is exerted directly over

the pont of tendernessC. flexion-extension of the right lower extremityD. asking the patient to cough

Schwartz 7th ed., p. 1385 / Ans. A

165. In children with history of URTI 3 days prior to development of RLQ pain, the differential diagnosis most often confused with appendicitis is;

A. meckel’s diverticulitisB. perforated peptic ulcerC. acute gastroenteritisD. acute mesenteric adenitis

Schwartz 7th ed., p. 1387 / Ans. D

166. The recommended treatment for patients with adenocarcinoma of the appendix is;A. appendectomyB. right hemicolectomyC. total colectomyD. subtotal colectomy

Schwartz 7ht ed., p. 1392 / Ans. B

167. The precipitating factor in secondary omental torsion is;A. bifid omentumB. accessory omentumC. obesityD. foci of intraabdominal inflammation

Schwartz 7th ed., p. 1556 / Ans. D

168. The most common solid tumor of the omentum is;A. lymphomaB. liposarcomaC. leiomyosacomaD. metastatic carcinoma

Schwartz 7h ed., p. 1558 / Ans. D

169. The most common anaerobic bacteria that is isolated in intraabdominal infection is;A. clostridium speciosB. peptococcus speciosC. bacteroides speciosD. fusobacterium specios

Schwartz 7th ed., p. 1524-1525 / Ans. C

170. A 45 year old female came in because of right upper quadrant pain, colicky aggravated by fatty food intake 4 hours ptc. Physical exam showed a palpable mass at the right

Page 29: Surgery- board exam questions

subcostal margin non-movable tender, midinspiratory arrest was illicited while the palpating hand was on the right subcostal area . the most likely condition is/are

A. acute appendicitisB. acute cholecystitisC. perforated peptic ulcerD. acute ascending cholangitis

Schwartz / Ans. B

171. The laboratory examination of a 45 year old female came in with the following results ; sgpt was elevated, direct bilirubin increased, alkaline phosphatase increased, protime prolonged . utz showed a hyperechoic mass at he neck of the gallbladder with posterior shadowing. The most likely diagnosis is

A. acute appendicitisB. acute infectious hepatitisC. acute calculous cholecystitisD. ascending cholangitis

Schwartz / Ans. C

172. The treatment of choice for the above condition ( no.4) isA. appendectomyB. medical managementC. cholecystectomyD. ercp

Schwartz / Ans. C

173. A patient s/p cholecystectomy and cbde with t-tube choledochostomy showed a distal cbd stone on t-tube cholangiography , management would be

A. daily irrigation with nssB. ercpC. choledococopy and basket extraction of the stoneD. repeart explore lap and cbde

Schwartz / Ans. C

174. A 25 year old male came in with chronic draining lesion at the perianal area of 2 years duration. Physical exam showed a draining lesion about 4 cm from the anal verge at the 3:00 position. The internal opening is probably

A. anterior midlineB. posterior midlineC. radially directly oppositeD. variable

Schwartz / Ans. B

175. 30 year old female came in because of painfull perianal mass 2 days ptc, physical exam showed a tender mass at the right lateral position w/c precludes rectal exam with a bulging tender mass adjacent to it. The most likely condition is

A. thrombosed external hemorrhoidsB. prolapsed internal hemorrhoidC. perianal abscessD. rectal polyp

Schwartz / Ans. C

176. A 50 year old male came in because of loss of weight about 30% of his previous body weight. Associated with on and off colicky abdominal pain. He also noticed that his stools have diminish in caliber just like a goat stool,the single most important examination would be

A. fecalysis B. ba enema

Page 30: Surgery- board exam questions

C. colonoscopy w/ biopsyD. ct scan

Schwartz / Ans. C

177. A 25 year old female came in with a palpable breast mass noted 2 weeks ptc, the mass is about 2.5 cm well delineated border, movable, non-tender at the right upper outer quadrant, no familial history of ca.the most likely diagnosis would be

A. fibroadenomaB. fibrocystic disease C. breast caD. galactocoele

Schwartz / Ans. A

178. Seromas are fluid collections coming from the:A. PlasmaB. InterstitiumC. LymphD. Capillary oozing

179. A patient was discharged 3 days after appendectomy (suppurative). After 5 days, he came back at the OPD for follow up complaining of tenderness at the operative site. The incision site was nonerythematous. What is your diagnosis?

A. HyperesthesiaB. Surgical site infectionC. Foreign body reaction to sutureD. Urinary tract infection

180. A patient was operated on for acute cholecystitis. After 24h postop, the patient developed fever. What is the most likely cause of his fever?

A. Cathether sepsisB. Drug-related conditionsC. Aspiration pneumoniaD. Atelectasis

181. The basic problem in poor wound healing regardless of the underlying factor is:A. Poor local hemostasisB. AnemiaC. Low 02 tensionD. Impaired inflammatory response

182. Ileus following abdominal surgery is expected to last for at least how many days?A. 3 B. 4 C. 5 D. 6

183. For every degree rise in temperature, the insensible water loss in cm3 per day is approximately:

A. 100 B. 150 C. 200 D. 250

184. Which ion if altered determines the shift of fluid from one compartment to another?A. SodiumB. PotasiumC. Chloride

Page 31: Surgery- board exam questions

D. Bicarbonate

185. Which of the following cytokines may help control keloids and hypertrophic scars?A. PdgfB. EgfC. TNFD. TGB-B

186. Which part of the GIT provides strength in the anastomosisA. SerosaB. MuscularisC. SubmucosaD. Mucosa

187. Which amino acid provide energy source in the GITA. AlanineB. ValineC. GlutamineD. Tryptophan

188. A 54yo male patient with 3rd degree burn approximately 40%TBSAdeveloped abdominal pain on the 4th hospital day. What is the least likely cause of the abdominal pain?

A. Acalculous cholecystitisB. Acute pancreatitisC. Superior mesenteric artery syndromeD. Renal colic

189. Convulsions if present during the resucitative phase of burn injury may be due to A. HypoxemiaB. HyponatremiaC. InfectionD. Hypokalemia

190. The offending organism in surgical site infection is;A. Staph aureusB. PseudomonasC. E. ColiD. Bacteroides

191. Earliest manifestations of catheter related complication following parenteral nutrition.A. TachycardiaB. FeverC. Glucose intoleranceD. Changes in sensorium

192. Which of the following statements is not a sound principle in the fluid and electrolyte therapy post operatively?

A. Thorough evaluation of the pre and intra-op fluid statusB. Urine volume is replaced on a mililiter to mililiter basisC. It is not necessary to give potassium with in the first 24 hourD. Insensible water loss is considered in the fluid therapy

193. The average potassium (in meq/l) content of the bile per day isA. 5B. 10C. 15D. 18

Page 32: Surgery- board exam questions

194. Which cranial nerve is involved in the act of swallowingA. XiB. XC. XiD. Viii

195. Primary reason for staging esophageal cancer isA. Determine its resectabilityB. PrognosticationC. To assess whether the procedure is for cure or palliationD. Whether pre-op chemotherapy is indicated

196. Most common benign esophageal tumor isA. LeiomyomasB. FibromasC. MyomasD. Fibromyomas

197. Blood supply of lesser curvature isA. Left gastricB. Right gastricC. GastroepiploicD. Splenic

198. Manifestation of acute gastric dilatationA. PallorB. Rapid respirationC. Changes in sensoriumD. Hypotension

199. A patient underwent upper gastrointestinal endoscopy for gastric outlet obstruction. The endoscopist noted a lesion at the antrum and took biopsy. It turned out to be gastric cancer. What type of gastric cancer has favorable prognosis?

A. Superficial spreadingB. PolypoidC. UlcerativeD. Linitis plastica

200. A patient diagnosed to have perforated peptic ulcer disease had his symptoms 8 hours prior to admission. He was prepared for surgery. Which of the following best describes the required operation?

A. Repair of perforation and selective vagotomyB. Repair of perforation and truncal vagotomyC. Suture of perforationD. Omental patch

201. After truncal vagotomy for peptic ulcer disease, the patient may develop the following except:

A. Gastric stasisB. Megaloblastic anemiaC. DiarrheaD. Gallstone formation

202. Most common neoplasm of the stomachA. AdenocarcinomaB. Lymphoma

Page 33: Surgery- board exam questions

C. LeiomyosarcomaD. Squamous cell carcinoma

203. A 77 year old, male, chronic smoker on NSAIDs for arthritis presents with an acute abdomen and pneumoperitoneum. What could be the most likely cause of this?

A. Severe abdominal pain due to chronic gastritisB. Acute pancreatitisC. Acute cholecystitis with cholangitisD. Perforated peptic ulcer disease

204. Increased venous pressure , decreased pulse pressure and decrease heart sound are pathognomonic of:

A. Acute myocardial infarctionB. PneumothoraxC. Cardiac tamponadeD. Aneurysm of the arch of aorta

205. “Bird’s beak” deformity is a radiologic sign of A. Sigmoid volvolusB. Ileoascending intussuceptionC. Sigmoid carcinomaD. Cardiac tamponade

206. A 32 yo male was brought to the ER because of blunt abdominal trauma due to fall. The patient had hematoma and abrasions on his RUQ and R flank. Your resident assessed that the patient has surgical abdomen and needs immediate surgery. He requested one-shot intravenous pyelography en route to or. The reason for such procedure is to:

A. Find out extravasation of urineB. Localize site of obstructionC. Determine function of either kidneysD. Quantify volume of urine output

207. A 23 yo female, medical student, was brought to the ER because of VA. Patient was GCS 13-14 with blunt abdominal trauma following a motor vehicular accident. Diagnostic peritoneal lavage was done & showed negative results. The assessment of negative result signifies:

A. No hollow viscus injuryB. Absent blood in peritoneal cavityC. Needle did not reach the peritoneal cavityD. Does not rule out abdominal organ injury

208. The most important factor in the management of contaminated wounds isA. Broad spectrum antibiotisB. Anti-tetanus prophylaxisC. Adequate debridementD. Closure without tension

209. Whether it’s diverticulization or exclusion, the principle behind these procedures for the doudenal injuries is to

A. Prevent fistula formationB. To rest or isolate the injure doudenumC. Prevent leakageD. Contain the inflammation at the RUQ

210. Sphincterotomy for anal fissures is usually done at theA. AnteromidlineB. Posterolateral

Page 34: Surgery- board exam questions

C. Postero midlineD. Anterolateral

211. Surgical management of familial adenomatous polyposis depends on the A. Number of polyps seenB. Size of polypsC. Histologic gradingD. Staging

212. Elective colorectal cancer operation is classified asA. CleanB. ContaminatedC. Clean contaminated D. Dirty

213. Which of the following is the most important prognostic determination of survival after treatment of colorectal cancer

A. Lymph node involvementB. Transmural extensionC. Tumor sizeD. Histologic differentiation

214. A 60 y.o. male is admitted to the hospital after passing out large amount of maroon-colored stools. At the ER, he again passed out more bloody stools as well as clots. He is pale and tachycardic, NGT aspirate is bilious. After resuscitation, which of the ff is the most appropriate initial test:

A. Barium enemaB. Rigid proctoscopyC. ColonoscopyD. RBC tag scan

215. Which is the most important stimulus for triggering endocrine response to injuryA. HypovolemiaB. Afferent nerve stimulation from the injured areaC. Tissue acidosisD. Local wound factors

216. Which is not true regarding gastrostomy tube feeding?A. Can be done endoscopically or open methodB. Indicated for mentally obtunded patientsC. Caloric requirement is based on patient’s needD. preferably delivered in a constant flow

217. Pellagra, dermatitis, glossitis, & peripheral paresthesias with spinal cord symptoms are due to deficiency of

A. Folic acidB. Vitamin aC. Pantothenic acidD. Zinc

218. The primary source of energy following trauma is A. GlycogenB. LipidsC. ProteinD. Carbohydrates

219. Eicosanoids are cell mediators derived from

Page 35: Surgery- board exam questions

A. GlycerolB. CholesterolC. Arachidonic acidD. Glutamine

220. The reason why approximately one half of breast cancer are located at the upper outer quadrant is

A. Its lymphaticsB. The predeliction of cancers superiorlyC. The larger volume of breast tissueD. Abundance of blood supply

221. Which structures demarcates the node levels in breast cancer:A. Latissimus dorsiB. Pectoralis majorC. Axillary arteryD. Pectoralis minor

222. Carcinoma of the breast among the elderly presenting as a bulky, colloid tumor isA. MucinousB. MedullaryC. AdenoidD. Apocrine

223. Management of inflammatory breast carcinoma with the best responseA. Neoadjuvant chemo + MRM+ irradiationB. Classical mastectomy + irradiationC. Chemotherapy+irradiationD. MRM+hormonal manipulation

224. Which of the following structures which when encroached by cancer gives rise to the “orange-peel” appearance of the breast?

A. glands of MontgomeryB. Ductal ampullaC. LobulesD. Subdermal lymphatics

225. A 34 female patient came at the OPD with 2cm palpable mass, moveable, nontender. No axillary nodes noted. Patient has an elder sibling who died of breast ca last 2 years ago. What will be your initial diagnostic procedure?

A. Needle or open biopsyB. MammographyC. Open biopsy ultrasound of the breastD. Magnetic resonance imaging

226. Collagen synthesis is at its peak at around how many days?A. 1-2 B. 3-5 C. 5-7 D. 10-14

227. Formation of collagen fibers by cross-linking is enhanced by lysyl oxidase together withA. MagnesiumB. CopperC. ZincD. Chromium

Page 36: Surgery- board exam questions

228. Which of the following steroids useful in the treatment of scars and keloidsA. BetamethasoneB. TriamcinoloneC. PrednisoloneD. Cortisone

229. Most common cause of hospital acquired infections are coming from the A. Respiratory tractB. IntraabdominalC. Urinary tractD. Blood

230. 45 male came in to the ER with a 3 cm lacerated wound at the left leg after he was bitten by a pig. He has received tetanus toxoid and tetanus immunoglobulin 10 1/2 years ago. What would be the appropraite management at the ER?

A. Toxoid + tetanus immunoglobulin+antibioticsB. Tetanus immunoglobulin onlyC. Debridement, leave wound open + tetanus immunoglobulin

+toxoid+antibioticsD. Tt+ats+cloxacillin

231. Superior vena caval obstruction is commonly caused byA. Fibrosing mediastinitisB. Bronchogenic carcinomaC. Caval thrombosisD. Aortic aneurysm

232. The average length of the adult trachea isA. 8 cmB. 9 cmC. 10 cmD. 11 cm

233. Squamous cell ca of the lung is usually associated withA. HypercalcemiaB. HyperkalemiaC. HyperglycemiaD. Hypomagnesemia

234. The initial diagnostic tool in assessing biliary ductal obstructionA. Alkaline phosphataseB. UltrasonographyC. ErcpD. Ct scan

235. Progressive painless jaundice with positive “ Courvoisier’s sign” suggestA. CholedocholithiasisB. Hydrops of gall bladderC. Pancreatic head or peri ampullary neoplasmD. Choledochal cyst

236. The initial diagnostic procedure in patients presenting with lower GI bleedA. NGT intubationB. ProctosigmoidoscopyC. Barium enemaD. Colonoscopy

Page 37: Surgery- board exam questions

237. The main parameter for determining the clinical stage of head and neck tumors isA. Size of primary tumorB. Lymph node involvementC. MetastasisD. Functional loss

238. Malignancy of the tongue usually occurs at theA. TipB. MidlateralC. CentralD. Posterior

239. The second most frequent benign salivary gland neoplasm isA. Pleomorphic adenomaB. Oxyphilic adenomaC. Papillary cystadenoma lymphomatosumD. Sialadenoma

240. The acidosis in hypovolemic shock is due to A. PhosphateB. Lactic acidC. Carbonic acidD. HCL

241. Substance associated with increased incidence of mesotheliomaA. SmokingB. SilicaC. AsbestosD. Lead

242. Prostatic carcinoma usually metastasizes to theA. LiverB. KidneysC. BladderD. Bone

243. The earliest visible evidence of neoplastic transformation isA. HyperplasiaB. MetaplasiaC. DysplasiaD. Angiogenesis

244. Epstein-barr virus is associated with the following tumors, exceptA. Burkitts lymphomaB. Nasopharyngeal carcinomaC. Hodgkin’sD. Hepatocellular carcinoma

245. A normal to low CVP that does not rise with rapid administration of crystalloids indicates:

A. Inadequate rate of infusionB. Improper placement of CVPC. Continuing hypovolemiaD. Improper choice of crystalloids

246. Recurrent laryngeal nerve is preserved in thyroid surgery. What is false with regards to this nerve:

Page 38: Surgery- board exam questions

A. Injury to both nerves cause upper airway obstructionB. Injury to the ipsilateral nerve cause disability of phonationC. It is a branch of cranial nerve xD. 64% of the r recurrent is seen along the tracheosophageal groove

247. Among the types of thyroid cancer, which one is the most aggressive?A. Follicular B. Papillary C. Hurthle D. Anaplastic

248. A 45yo female patient underwent total thyroidectomy for follicular ca. 24 H after surgery, the patient developed numbness around the lips and fingers. What could have caused this?

A. Transection of recurrent nerveB. Ischemia to the parathyroidsC. HematomaD. Transection of superior laryngeal nerve

249. True regarding hernia:A. Most common type of hernia in females is direct herniaB. Strangulation is more common in indirect than in femoral herniaC. Tension-free repair is best achieved with bassini techniqueD. Repair of umbilical hernia is indicated in infants with hernia > 2cm

250. A 23yo male patient came in at ER with irreducible inguinal hernia of >16hours duration. Patient complained of abdominal pain with BP of 100/70, CR 120, & RR 30. The appropriate management is:

A. Observe for progression of painB. Sedate patient and reduce the herniaC. Apply ice pack over the herniaD. Schedule patient for surgery

251. At the ER, a patient complained of renal colic. The following are the minimal laboratory evaluation you would request except:

A. CBC and electrolytesB. IVPC. Bun and creatinineD. Urinalysis

252. In the medical treatment of BPH, which among the drugs acts by reducing the intraprostatic dihydrotestosterone levels without lowering the plasma testosterone level?

A. DoxazosinB. FinasterideC. TerazosinD. Tamsulosin

253. A 48hour old baby boy was brought to the ER due to vomiting of bilious vomitus, abdominal distention, and failure to pass out meconium. What is the most likely cause?

Tracheo-esophageal atresiaIleal atresiaPyloric stenosisRectal mass

254. A 28 kg child with ruptured appendicitis has the following laboratory results: BP=90/60; PR=110; RR=25; Na=125; Cl=95; K=3; ph=7.35; pCO2=35; pO2=100; HCO3=24. Which of the following is TRUE?

Page 39: Surgery- board exam questions

A. anion gap is increased B. uncompensated metabolic acidosis is present C. actual Na deficit is 250 mEq D. the ideal intravenous fluid to infuse is D50.3%NaCl

255. Interpret the above ABG results:A. compensated respiratory alkalosisB. uncompensated metabolic acidosis C. compensated metabolic acidosis D. uncompensated metabolic alkalosis

256. In the above patient, which of the following is TRUE? plasma volume is 5,600 cc initial fluid resuscitation is 560 cc given as fast drip decrease in Na is primarily due to renal losses decrease in Na is dilutional in nature

257. In the same patient, which abnormality is he liable to manifest?A. increased intracranial pressure B. dry sticky mucous membranes C. shift of oxygen-hemoglobin dissociation curve to the left D. ventricular arrhythmias

258. Extracellular fluid deficit results in:A. increased sodium B. increased hematocrit C. decreased BUN D. decreased creatinine

259. A 70-kg is admitted because of abdominal colic followed by vomiting of previously ingested food. Initial assessment is an Intestinal Obstruction probably from adhesions. On examination, the patient has weak and rapid pulse, depressed eyeballs, and dry tongue and mucous membranes. The most likely fluid state of this patient is:

A. hypotonic dehydration B. isotonic dehydration C. hypertonic dehydration D. hypernatremia

260. TRUE statements regarding fluid and electrolyte losses in this patient: A. vomiting accounts for an insignificant amount of the volume loss B. can go into hypovolemic shock anytime C. sequestration of fluid in the urinary system accounts for majority of the losses D. compensatory fluid shift from other compartments is sufficient to maintain

homeostasis

261. Fluid replacement in this patient should begin with: A. blood C. crystalloids B. colloids D. plasma

262. Which of the following is an expected cellular change/s in hypovolemic shock? A. intracellular gain of Na+ and H20 B. depletion of cellular ATP C. extracellular increase of K+ D. increase in transmembrane potential

Page 40: Surgery- board exam questions

263. A patient has a blood pressure of 70/50 mmHg and a serum lactate level of 30-mg/100 ml (normal: 6 to 16). His cardiac output is 1.9 L/min, and his central venous pressure is 2 cmH2O. The most likely diagnosis is:

A. congestive heart failure C. cardiac tamponade B. hypovolemic shock D. pulmonary embolus

264. During compensation for hypovolemic shock, blood flow to which of the following area/s is under sympathetic nervous system control?

A. brain C. small bowel B. heart D. liver

265. This hemodynamic monitor is valuable to differentiate a pericardial tamponade from hypovolemia:

A. CVP C. pulmonary catheter B. Arterial catheter D. capnograph

266. In a multiply injured patient with acute blood loss, adequate preload to the heart is maintained initially by the:

A. increase in systemic vascular resistance B. development of tachycardia C. hormonal effects of angiotensin D. hormonal effects of antidiuretic hormone

267. An example of neurogenic shock: A. convulsion C. carpopedal spasm B. syncope D. involuntary fine tremors

268. Coagulation factors tested by prothrombin time (PT): A. VII C. VIII B. XII D. XI

269. The initial hemostatic response to injury is: A. platelet formation C. vascular constriction B. hemolysis D. initiation of coagulation cascade

270. All are major events in the hemostatic process: A. vascular dilatation C. fibrin deposition B. platelet plug formation D. hemolysis

271. The following are suggestive of DIC: A. platelet counts of 250,000/cu.mm C. normal fibrinogen level B. increased fibrin split products D. hypofibrinogenemia

272. Injury to the intima of blood vessels exposes the subendothelial collagen which initiates: A. fibrinolysis C. release of cyclic AMP B. platelet aggregation D. vasoconstriction

273. A 77-year old man is scheduled to undergo sigmoid colectomy. He denies any history of prolonged bleeding. Preoperative evaluation of hemostasis should include:

A. no screening tests B. prothrombin time (PT) and partial thromboplastin time (PTT) C. platelet count, blood smear, and PTT D. platelet count, PT, and PTT

274. The total caloric requirement of a malnourished patient is calculated using his: A. Actual weight C. 50% of ideal body weight B. 75% of ideal body weight D. Ideal body weight

Page 41: Surgery- board exam questions

275. In prescribing the nutrition therapy, micronutrients are given at what % of RDA: A. 100 C. 60B. 80 D. 50

276. Nitrogen excretion is less in: A. starvation C. severe burns B. sepsis D. elective surgery

277. At the initial stages of septic shock it is normally expected to have a: A. normal blood volume C. cold extremities B. hypertension D. hypothermia

278. A 38-year-old male had abdominal exploration for multiple gunshot wounds. He is febrile T 38.5 C on the 2nd post-op day; HR is 98 bpm, BP 100/80. WBC count is 13000. The patient is hooked to a ventilator. The patient is developing what condition;

A. SIRS C. severe sepsis B. sepsis D. septic shock

279. On the 5th day post-op the patient in addition to the above findings now shows erythema and draining pus from the abdominal incision site. The patient now is developing;

A. SIRS C. severe sepsis B. sepsis D. septic shock

280. On the 10th day post-op, the patient is persistently febrile with increase in WBC count (18000) and this time with oliguria that is unresponsive to fluid resuscitation. The patient now is developing;

A. SIRS C. severe sepsis B. sepsis D. septic shock

281. The above patient was given pressor agents to improve his condition, despite this he remains oliguric. The patient now has:

A. SIRS C. severe sepsis B. sepsis D. septic shock

282. Immediately after mastectomy your patient asks if any of the following promotes wound healing, which one wills you advise your patient?

A. Vitamin C C. Vitamin BB. Zinc supplementation D. carbohydrate rich foods

283. Severe cases of hidradenitis suppurativa in the groin area are best managed by excision of the involved area and:

A. closure by secondary intention B. delayed primary closure C. primary closure D. partial thickness skin grafts

284. An elderly cancer patient fell and sustained a deep lacerated wound over the right forehead about 9 cm length. If wound infection later develops in this patient. The major cause of impaired wound healing is:

A. anemia C. DM B. local wound infection D. all of the above

285. What technique of wound closure is recommended in a patient with ruptured appendix with spreading peritonitis?

A. primary C. secondary B. delayed primary D. tertiary

Page 42: Surgery- board exam questions

286. If the lacerated wound is sutured closed, this healing is known as; A. primary C. tertiary B. secondary D. delayed primary

287. Wound healing in this patient may be impaired because of his; A. age C. type of wound B. depth of the wound D. cancer

288. Infection in the above wound is partly controlled by the presence of what cells; A. macrophages C. endothelial cells B. fibroblast D. T lymphocytes

289. Important activities of macrophages during wound healing; A. wound debridement C. activation of coagulation cascade B. arginine synthesis D. fibrin clot

290. The ABC’s of resuscitation includes the following: A. establish adequate airway B. bleeding should be controlled by applying tourniquet C. circulation should be maintained by rapid infusion of blood D. oxygen should be delivered if necessary

291. A 25-year-old male is brought to the ER after he sustained a stab wound to the 5th intercostals space. PE: conscious, stretcher-borne, vitals are as follows: blood pressure 90/50, pulse 110/min and respiratory rate 30/min. Which of the following statements is true:

A. primary survey and initial resuscitation should be instituted one after the other B. a complete detailed physical exam should be performed so as not to miss any

injury C. large-bore IV lines should be inserted D. blood for CBC and x-matching should be drawn

292. An 8-year old boy has a closed angulated deformity of his right forearm. VS: BP = 99 mm Hg systolic and PR = 110 Identify the most emergent problem of the victim. Write any of the following:

A. Airway or Cervical Spine C. Circulation B. Breathing D. Disability or Neurologic Injury

293. A 7-year old boy is gasping for breath. He can speak clearly, but complains of chest pain and an inability to get his breath. He is becoming increasingly anxious. VS: BP = 100 mm Hg systolic and PR = 110 Identify the most emergent problem of the victim. Write any of the following:

A. Airway or Cervical Spine C. Circulation B. Breathing D. Disability or Neurologic Injury

294. A 25-year-old man is unconscious, withdraws from painful stimuli, eyes are closed, but they open in response to pain, and he is making unintelligible sounds. His pupils are equal, and both react sluggishly to light. His Glasgow coma scale score is:

A. 5 C. 11 B. 8 D. 14

295. After control of this patient's airway is achieved, the first diagnostic study to obtain is: A. x-rays of the skull C. x-rays of the cervical spine B. CT scan of the head D. carotid angiography

Page 43: Surgery- board exam questions

296. A 19-year-old woman presents with a non-bleeding stab wound, 1 cm long, in the anterior triangle of her neck, 3 cm above her left clavicle. Examination of the neck reveals that the wound has penetrated the platysma muscle, and that the left carotid pulse is normal; there is no hematoma or bruit. Appropriate management of this patient:

A. immediate tracheostomy B. nasogastric intubation C. chest x-rays D. formal neck exploration under general anesthesia

297. Following an automobile accident, a 30-year-old woman is discovered to have a posterior pelvic fracture. Hypotension and tachycardia respond marginally to volume replacement. Once it is evident that her major problem is free intraperitoneal bleeding and a pelvic hematoma in association with the fracture, appropriate management would be:

A. application of antishock trousers and exploratory laparotomy B. arterial embolization of the pelvic vessels C. exploratory laparotomy and possible bilateral internal iliac artery ligation D. external pelvic fixation to stabilize the pelvis

298. Large amount of free intraperitoneal blood is seen in a patient with penetrating abdominal injury after an abdominal ultrasound, the next best step is to:

A. apply antishock trousers and exploratory laparotomy B. arterial embolization of the pelvic vessels C. exploratory laparotomy D. external pelvic fixation to stabilize the pelvis

299. A 6-year-old is having episodes of watery diarrhea for the past three days. She is lethargic, irritable and restless. The heartbeat is rapid but the pulses are weak. The child should:

A. stay home and call the pediatrician for advice B. stay home and have a cold bath C. proceed to the nearest hospital emergency room D. get a glass of cold fruit-juice to drink

300. Over-enthusiastic fluid resuscitation should be avoided in this child because of: A. potential liver failure B. acute myocardial infarction C. venous thrombosis and rupture D. potential cerebral injury

301. A study on ruptured appendicitis will be conducted in a tertiary university hospital that includes all age groups and all genders on the rate of SSI between primary vs. delayed primary closure. What is the possible confounder in this study?

A. all age groups B. ruptured appendicitis C. SSI D. primary vs. delayed primary

302. The most common type of inguinal hernia in both males and females is: A. Indirect B. Direct C. Femoral D. Mixed.

303. The Cooper’s ligament repair for direct or femoral hernia is popularized by:A. Bassini B. McVay C. Halsted D. Ferguson

304. A 22 year old female consulted a physician because of a 2x2x2 cms, movable, well circumscribed, non tender mass, noted accidentally while taking a bath. Physical examination was consistent with the findings above. Most likely diagnosis is:

Page 44: Surgery- board exam questions

A. Breast Ca B. Fibrocystic change C. Fibroadenoma D. Phylloides tumor

305. The next thing to do on the above patient is: A. Reassurance and observation B. Ask for a mammogram C. Partial mastectomy D. Hormonal treatment

306. Ivory an 18 year old female consulted because of breast tenderness becoming more severe just before menses. PE – showed asymmetric nodularities on the upper outer quadrant of both breasts with no dominant mass . Menstrual cycle is monthly and regular. Most likely diagnosis is:

A. Sclerosing adenosis C. Fibrocystic change B. Normal premenstrual tension D. Breast Ca

307. Sarah Jane a 35 year old female consulted because of an intermittent bloody nipple discharge on the right breast PPE- no mass noted on the right breast Most likely diagnosis is:

A. Papillary cancer C. Bleeding galactocoele B. Mammary duct ectasia D. Intraductral papilloma

308. Chemotherapy in breast carcinoma is: A. given to those with at least 4 positive axillary metastasis B. usually a combination of multiple agents C. free of any side effects D. used to control local recurrence

309. Breast malignancy that is known to have the highest incidence of bilaterality and multicentricity:

A Paget’s disease of the breast C. Papillary carcinoma B. Lobular carcinoma D. Medullary carcinoma

310. The next appropriate thing to do in managing the above case will be: A. Local wound exploration B. CT Scan C. Immediate surgery D. Triple angiography of the abdomen

311. Fifteen minutes later the BP became 60 palpatory, PR-120/min accompanied by abdominal distention. You will recommend:

A. CT scan C. MRI B. proceed with local wound exploration D. Immediate surgery

312. The most commonly injured intraabdominal organ following a blunt abdominal trauma; A. Pancreas B. spleen C. small intestine D. colon

313. A 55 y/o male, smoker underwent FNAB for a 4cm preauricular mass. Results showed (+) for malignant cells .The patient is most probably suffering from:

A. Pleomorphic adenoma C. mucoepidermoid carcinoma B. Follicular carcinoma D. papillary carcinoma

314. The appropriate treatment for a benign parotid neoplasm is: A. “shelling out” of the tumor C. total parotidectomy B. superficial parotidectomy D. radiation therapy

315. Most common histology of head-neck cancers is: A. Adenocarcinoma C. squamous cell carcinoma

Page 45: Surgery- board exam questions

B. papillary carcinoma D. Adenosquamous carcinoma

316. The most common causative agent being implicated in head-neck cancers is: A. UV radiation B. Tobacco C. nitrates D. alcohol

317. The site of injury corresponds to which zone of the neck? A. Zone I C. Zone III B. Zone II D. Zone IV

A 60 year old woman with chronic renal failure and undergoing hemodialysis 2-3x/week develops tumoral calcinosis. Her parathyroid hormone is markedly elevated while her Ionized Calcium is normal.

318. The diagnosis of the above is: A. Primary Hyperparathyroidism B. Secondary Hyperparathyroidism C. Tertiary Hyperparathyroidism D. Parathyroid carcinoma.

319. The course of papillary thyroid cancer is best described by which of the following statements?:

A. Metastases are rare; local growth is rapid B. Local invasion and metastases almost never occur C. Metastases frequently occur to cervical lymph nodes, distant metastases

and local invasion are rare D. Rapid widespread metastatic involvement of the liver ,lungs and bone results in

a 10% 5 yr survival rate

320. The most important prognosticating factor in thyroid cancer is: A. tumor size C. lymph node metastases B. patient’s age D. histologic grade

321. 43 years old female present with diffuse enlargement of the thyroid gland accompanied by palpitation, heat intolerance and weight loss. She is most probably suffering from :

A. Follicular Cancer C. Grave’s diseaseB. Plummer Vinson’s disease D.Colloid goiter

322. A 50 years old male has just undergone a total thyroidectomy for follicular cancer of the thyroid. On the third hour post-op, he experienced circumoral numbness. He is probably suffering from

A. Frey’s syndrome C. hypocalcemia B. hypothyroidism D. hypokalemic acidosis

A 16 year old high school student came to you with a complaint that her friends took notice of an anterior neck enlargement since she started on her weight reduction program. You were indecisive as to whether there really was a thyromegaly. The rest of the P.E. were normal

323. Which would be most helpful in determining whether there really is a thyromegaly or none:

A. a second opinion from an endocrinologist B. MRI of the neck C. Thyroid Scanning D. serum TSH, T3 & T4

324. A 72-year old male consults because of change in bowel habits for the last 2 weeks. On rectal exam, a hard, irregular mass was noted at 6 cm from the anal verge The most likely diagnosis here is:

Page 46: Surgery- board exam questions

A. Rectal carcinoid C. Rectal PolypB. Rectal carcinoma D. Anal canal carcinoma

325. The most appropriate initial diagnostic procedure to be done is: A. Ultrasound of the abdomen C. Chest x-ray B. Proctosigmoidoscopy D. CT scan of the pelvis

326. Majority of peptic ulcer patients are successfully managed medically or conservatively. Which of the following complications is the most common indication for surgical intervention?

A. Perforation C. Gastric outlet obstruction B. Massive GI bleeding D. Malignant degeneration

327. A 48 year old company manager who has been taking proton pump inhibitor irregularly for the past 2 years because of on & off epigastric discomfort presents in the emergency room with hematemesis and melena. This patient should be:

A. Aggressively resuscitated with fluids and measures to localize site of bleeding should be done in preparation for possible surgery

B. Explored immediately because bleeding is an indication for surgery C. Give massive doses of anti-ulcer medications D. Watchful waiting for 24-48 hours.

328. A 43 year old male patient with on & off epigastric pain during the past 15 months complains of sudden severe epigastric pain later on becoming generalized. An upright chest file taken at the ER shows a strip of lucency underneath the right hemidiaphragm. This patient has:

A. Penetrating peptic ulcer B. Acute pancreatitis C. Perforated diverticulitis D. Perforated peptic ulcer.

329. Above patient is best managed by: A. Aggressive medical treatment of PUD B. Laparotomy after resuscitation / IV antibiotics C. Fluid resuscitation, nasogastric decompression and somatostatin D. Emergency ERCP

330. A 30 y/o male who usually has dripping fresh blood after defecation suddenly had anal pain and a smooth, tender, 1 cm diameter rounded mass coming out of the anus. This is most likely:

A. peri-anal abscess B. internal hemorrhoids 3rd degree C. Anal wartD. thrombosed internal hemorrhoids

331. A 65 y/o female was referred for surgical consult because of abdominal pain and distention. Findings in the scout film of the abdomen that will be highly suggestive of intestinal obstruction:

A. Air in the distal rectum B. Pneumatosis intestinales C. luscency beneath the right hemidiapragm D. step ladder abnormality

332. A 75 y/o female is being worked up for paralytic ileus The electrolyte that is usually below normal is:

A. Potassium C. calcium B. Sodium D. Magnesium

Page 47: Surgery- board exam questions

333. A significant and independent influence on the prognosis of a patient who had an esophageal resection for cancer is:

A. Tumor penetration of the esophageal wallB. Cell type C. Degree of cellular differentiationD. Location of the tumor in the esophagus

334. Hepatic resection should be considered for localized metastatic spread from which of the following primary site?

A. colorectal C. Lungs B. stomach D. Breast

335. The standard of treatment for patient with biliary colic and cholecystolthiasis by ultrasound is

A. Cholecystectomy C. Stone Dissolution B. Cholecystostomy D. Endoscopic Stone removal

336. In an infected obstructed common bile duct producing cholangitis which of the following is the best option for drainage in a very poor risk patient ?

A. cholecystostomy B. operative CBD exploration and T tube choledochostomy C. ERCP and stenting D. MRICP

337. CT scan finding that is highly indicative of infected pancreatic necrosis; A. edema C. calcificationsB. dilated pancreatic duct D. gas bubbles in the retroperitoneum

338. When gallstone ileus occurs , obstruction is most frequent in : A. duodenum C. distal ileumB. jejunum D. Sigmoid colon

339. The most common cause of Acute Cholecystitis is : A. cystic duct obstruction B. E. coli infectionC. Gall bladder polyp near the neckD. multiple gallstones.

340. The strongest evidence of presence of gallstones by ultrasound is : A. Edema and thickening of gallbladder wall B. Contracted gallbladder C. Dilated extrahepatic bile ducts D. High level echoes with posterior acoustic shadowing.

A 45 year old jaundiced patient who has been diagnosed to have gallbladder and common bile duct stones by ultrasound 6 months ago is brought to the ER because of abdominal pain , chills, high grade fever and hypotension. PE: icteric sclerae , abdomen slightly tender with guarding RUQ.

341. The patient most likely is having which of the following: A. acute cholecystitis C. Acute suppurative obstructive cholangitis

B. empyema of the gallbladder D. ruptured gallbladder

342. The problem of patient in the previous number is best managed by: A. administration of oral antibiotics

Page 48: Surgery- board exam questions

B. careful observation and follow up C. surgical/endoscopic decompression of biliary tree after resuscitation and

antibiotics D. give systemic antibiotics and steroids.

343. The most likely diagnosis in a 70 y/o male presenting with significant weight loss accompanied by progressive jaundice, anorexia, pruritus and tea colored urine. PPE showed a palpable non tender gallbladder?

A. hepatoma C. pancreatic head cancerB. gallbladder cancer D. gallbladder empyema

344. The severity of pancreatitis, particularly in those patient who is not improving after 24 hours of medical management can be assessed using:

A. Angiography C. Dynamic CT Scan B. Scout film of the abdomen D. MRICP

345. Which of the following clinical clues is a reliable symptoms of arteriosclerosis obliterans?

A. Poor hair growth C. Cold hands and feet B. Deformed toenails D. Intermittent claudication.

346. What is the Most Common symptom of acute aortic dissection? A. Syncope C. Hemiparesis B. Severe pain D. Congestive heart failure.

347. This is the BEST procedure for a patient with acute head injury secondary to a vehicular accident:

A. CT scan C. Plain skull x-rays B. Angiography D. Electroencephalogram

348. Abrasion wounds on the face are best treated with:A. wound cleansing and debridement with normal salineB. split thickness skin grafting C. laser surgery D. all of the above are correct

349. Essential elements in the survival of a skin graft: A. Antibiotics C. Vascularity B. Dry dressing D. Nerve supply.

350. Most common cause of UTI in children:A. E. coli C. Klebsiella, Enterobacter B. Staph saprophyticus D. Proteus sp

351. A massively bleeding posterior duodenal ulcer, which of the following is most likely involved:

A. Left gastric arteryB. Gastroduodenal arteryC. Short gastric arteryD. Left gastroepiploic artery

352. Which of the following would inhibit parietal cell acid secretions?A. Histamine B. GastrinC. AcetylcholineD. Prostaglandins

Page 49: Surgery- board exam questions

353. The most common symptom associated with a Meckel’s diverticulum is:A. Crampy abdominal painB. Periumbilical painC. Blood per rectumD. Watery diarrhea

354. The typical visual field deficit caused by a pituitary adenoma with suprasellar extension is:

A. Bitemporal hemianopsiaB. Homonymous hemianopsiaC. Superior quadrantanopsiaD. Inferior quadrantanopsia

355. The evaluation of a comatose patient with a head injury begins with:A. Evaluation of cardiovascular systemB. Evaluation of pupillary reflexesC. Establishment of an airwayD. Plain film of the skull

356. The radial nerve is at greatest risk for injury with which fracture:A. Fracture of the surgical neck of the humerusB. Fracture of the shaft of the humerusC. Supracondylar fracture of the humerusD. Olecranon fracture

357. An early sign of compartment syndrome in the hand includes:A. Pain with passive stretch at the digitsB. Absent radial pulseC. Motor paralysisD. Swelling of the digits

358. The most common site of involvement for skeletal tuberculosis is the:A. TibiaB. FemurC. PelvisD. Spine

359. A 65-year old woman with a history of chronic constipation is transferred from a nursing home because of abdominal pain and marked abdominal distention. On examination, her abdomen is found to be distended and tender in the left lower quadrant. The most likely diagnosis is:

A. AppendicitisB. Carcinoma of the colonC. Volvulus of the sigmoid colonD. Small bowel obstruction

360. A 68-year old man presents with crampy abdominal pain and distention vomiting. Findings on PE are positive for healed abdominal scars. X-ray reveals multiple gas fluid levels. The WBC count is 12,000. The most likely diagnosis is:

A. Small bowel obstruction due to adhesionsB. A herniaC. AppendicitisD. Gallstones and ascites

361. A 56-year old man has suffered from intermittent claudication for 5 years. He has recently developed cramping abdominal pain that is made worse by eating. He has a history of a 15-lb weight loss. The most likely diagnosis is:

Page 50: Surgery- board exam questions

A. Chronic cholecystitisB. Chronic intestinal ischemiaC. Peptic ulcerD. Abdominal aortic aneurysm

A 28-year old man with a history of emotional disturbance enters the hospital with a history of weight loss and regurgitation of food. Regurgitation is worse when he lies down. The most likely diagnosis is:

A. Hiatal herniaB. Cancer of the esophagusC. Duodenal ulcerD. Achalasia

A 38-year old man with a history of fever associated with abdominal pain of several weeks duration presents now with a sudden onset of explosive abdominal pain and vomiting. Flat plate x-rays reveals air under the diaphragm. CT scan shows mesenteric lymphadenopathy and splenomegaly. The most likely diagnosis is:

A. Tuberculosis enteritisB. Typhoid enteritisC. Primary peritonitisD. Ulcerative colitis

Splenectomy is often indicated in the management of:A. Hereditary spherocytosisB. Hereditary neurofibromatosisC. Aplastic anemiaD. Hashimoto’s disease

A painless distended gallbladder palpable on PE of a jaundiced patient is strongly suggestive of:

A. Empyema of the gallbladderB. Mirizzi’s syndromeC. Gallstone impacted in the ampullaD. Pancreatic carcinoma

Which of the following suggest unresectability of a left upper lobe lung cancer:A. HemoptysisB. Malignant pleural effusionC. A cough specimen with positive sputum cytologyD. Clubbing and blueness of fingers

Which of the following studies is contraindicated in a drowsy patient with papilledema whom one suspects of having acute closed head trauma:

A. Carotid arteriographyB. Lumbar punctureC. CT scanD. Echoencephalography

The appropriate antibiotic for a patient with a cellulitis of a leg due to streptococcus is:A. PenicillinB. ErythromycinC. 3rd generation cephalosphorinsD. Cloxacillin

Massive bleeding from the lower GI tract (beyond the ligament of Treitz) is most often due to:A. DiverticulosisB. Meckel’s diverticulum

Page 51: Surgery- board exam questions

C. diverticulitisD. colonic carcinoma

A 54-year old male has been complaining of recurrent hypogastric pain for almost 6 months. For almost 3 weeks prior to consult, he had noticed that he is passing air during micturition. The most common cause of the above condition is:

A. carcinoma of the bladderB. tuberculosis of the urinary tractC. carcinoma of the sigmoid colonD. diverticulitis of the colon

A 42-year old woman has been complaining of intermittent gross hematuria. On urinalysis, she has significant pyuria but without bacilluria. The condition is most commonly due to:

A. urethritisB. bladder stoneC. perinephric abscessD. tuberculosis of the kidney

A 25-year old male had episodes of severe vomiting due to a gastric outlet obstruction. He appears pale and dehydrated. The likely metabolic abnormality the patient would have is:

A. hypochloremic acidosisB. hyponatremic acidosisC. hypochloremic alkalosisD. hypernatremic alkalosis

A 1week old infant presents with moderate respiratory distress and tympany in only one hemithorax. The most likely diagnosis is:

A. spontaneous pneumothoraxB. eventration of the diaphragmC. Bochdalek herniaD. Atelectasis

A little boy aspirated a peanut. He is cyanotic. On expiration, his mediastinum shifts to the right. The peanut most likely is lodged in the:

A. right main stem bronchusB. left main stem bronchusC. tracheaD. esophagus

A 40-year old man is hit by a car and sustains an injury to the pelvis. Which of the following is most indicative of a urethral injury:

A. hematuriaB. high-riding prostate on rectal examinationC. oliguriaD. scrotal ecchymosis

A 64-year old man has mild upper abdominal pain. On contrast CT-scan, a 5 cm lesion in the left lobe of the liver enhances and then decreases over a 10-minute period from without to within. The most likely lesion is a:

A. hepatic adenomaB. amoebic abscessC. fatty infiltrationD. carvernous hemangioma

A 28-year old male has had a nonseminomatous testicular cancer treated. In following this patient for possible recurrent tumor, the most useful serum marker would be:

A. carcinoembryonic antigen

Page 52: Surgery- board exam questions

B. alpha fetoproteinC. prostate specific antigenD. alkaline phosphatase

The most common cause of esophageal rupture or perforation:A. Endoscopic injuryB. Blunt chest traumaC. Boarhaave’s syndromeD. Carcinoma

The superior mesenteric artery communicates with the celiac artery via the:A. Pancreatico-duodenal arteryB. Splenic arteryC. Hepatic arteryD. Dorsal pancreatic artery

The most common offending organism in pyogenic osteomyelitis is:A. Escherichia coli B. Staphylococcus aereusC. Pseudomonas aeruginosaD. Entococcus

In patients who developed a documented episode of deep vein thrombosis, the most frequent and significant long term sequela is:

ClaudicationRecurrent foot infectionDevelopment of stasis ulcerPulmonary embolism

In an arterial stenosis, the most critical factor is:A. The length of the stenosisB. The cross-sectional area of the stenosisC. The length and area are equally importantD. Whether the stenosis is smooth or has an irregular countour

A 60-yr old man with a history of atrial fibrillation is found to have a cyanotic cold right lower extremity. The embolus most probably originated from:

A. An atherosclerotic plaqueB. Deep vein thrombosisC. LungsD. Heart

Which of the following elements is not a component of venous thrombogenesis:A. Incompetent valves in perforating veinB. Disruption of endothelial intimaC. VenostasisD. Hypercoagulability

In a patient suffering from an acute arterial embolic phenomenon, if the ischemia is not relieved, which of these following tissues is the first to be irreversibly damaged:

A. muscleB. nerveC. fatD. synovial membrane

Which of the following clinical manifestation may suggest tentorial herniation:

Page 53: Surgery- board exam questions

A. contralateral mydriasisB. decorticate rigidityC. contralateral hemiparesisD. ipsilateral hemianopsia

The most common complication of lung abscess is:A. EmpyemaB. PneumothoraxC. bronchopleural fistulaD. osteomyelitis of the ribs

The most common posterior-superior sulcus chest tumor in a 6-month old child is:A. NeuroblastomaB. TeratomaC. cystic hygromaD. ganglioneuroma

The most common cause of spontaneous pneumothorax is:A. tuberculosisB. emphysematous blebsC. pneumoniaD. bronchial rupture

What substance is released by macrophages in order to activate T helper/inducer (CD4+) lymphocytes:

A. Interleukin 1B. Interleukin 2C. Interleukin 3D. Interleukin 4

A 27-year old hypertensive, diabetic woman is admitted for cadaveric renal transplantation. She is blood type B and has had four transfusions of packed cells over the preceding 6 months. Which of the following factors would preclude transplantation:

A. Donor blood type OB. Positive crossmatchC. Two-antigen HLA match with donorD. Blood pressure of 180/100 mm Hg

The primary mechanism of action of cyclosporine A is inhibition of:A. Macrophage functionB. Antibody productionC. Cytotoxix T-cell effectivenessD. Interleukin 2 production

Which of the following malignancy exhibits a slow growing, late metastasizing characteristic:A. Squamous cell carcinomaB. Basal cell carcinomaC. MelanomaD. Basosquamous carcinoma

Following the Goodsall-Salmon’s rule in fistula in ano, an anteriorly located opening less than 3 cm from the anal opening follows:

A. Straight tractB. Curve tractC. No tractD. S-shaped tract

Page 54: Surgery- board exam questions

One of the following statements is correct regarding hyponatremia:A. Serum sodium determination would give a normal resultB. Hypoglycemia may be a causeC. Management could either be water restriction or replacement depending

on the causeD. Hyperactive deep tendon reflex is a manifestation

A 60 kg lean male would normally have this amount of body water:A. 36 litersB. 24 litersC. 30 litersD. 48 liters

A practical but useful clinical parameter that can be used to assess the adequacy of volume replacement in patients suffering from hypovolemia is:

A. Central venous pressure measurementB. Urine outputC. Blood pressureD. Pulse rate

A 42-year old man who has been on prolonged total parenteral nutrition (TPN) administration was noted to have scaly, hyperpigmented lesions over the acral surface of elbows and knees and alopecia. The most likely cause of the condition is:

A. Essential fatty acid deficiencyB. Excess glucose caloriesC. HypomagnesemiaD. Zinc deficiency

A 40-year old man is found to have severe metabolic acidosis with a high anion gap. 50-57The most likely cause is:A. DiarrheaB. Renal tubular acidosisC. UreterosigmoidostomyD. Methanol ingestion

A 60-year old female, post mastectomy for breast cancer, presents with headache backache and frequent vomiting. She is extremely thirsty and stuporous. The test most likely to identify the cause is:

A. Serum calcium determinationB. Serum sodium determinationC. Serum potassium determinationD. Serum glucose determination

The Trendelenburg’s (head down) position is beneficial in the initial management of which type of shock:

A. Hypovolemic shockB. Cardiogenic shockC. Septic shockD. Neurogenic shock

The most common form of burn is:A. scald burnsB. flame burnsC. chemical burnD. electrical burn

Page 55: Surgery- board exam questions

A 20 year old man has lymphadenopathy behind and inferior to his right ear. Biopsy shows the lesion to be a lymphosarcoma. The most likely site of the primary tumor is:

A. Floor of the mouthB. NasopharynxC. Buccal mucosaD. Mediastinum

The most frequent organism in highest density that can be isolated from contamination following colon perforation are:

A. Gram positive aerobesB. Gram negative aerobesC. Gram positive anaerobesD. Gram negative anaerobes

The most clinically significant early physiologic abnormality in post traumatic pulmonary insufficiency is:

A. Increased physiologic shuntB. Increased dead spaceC. Cardiogenic pulmonary edemaD. Pulmonary fibrosis

354. Overall bone mass increases up to what age afterwhich there is an overall decrease in bone mass:

A. 10 to 15 years of ageB. 15 to 20 years of age Answer: D; Schwartz 8th ed p.1654C. 20 to 25 years of ageD. 30 to 35 years of age

355. After multiple myeloma, the most common primary malignant bone tumor is:A. OsteosarcomaB. Ewing’s sarcoma Answer: A; Schwartz 8th ed p.1661C. ChodrosarcomaD. Fibrosarcoma

356. The best treatment for septic arthritis of the hip is:A. Repeated daily aspirationB. Arthroscopic drainageC. Surgical drainage/Arthrotomy with antibiotic therapyD. Antibiotic therapy only Answer: C; Schwartz 8th ed p1678

357. The most common form of inflammatory arthritis:A. OsteoarthritisB. Rheumatoid ArthritisC. Septic ArthritisD. Psoariatic Arthritis Answer: B; Schwartz 8th ed p.1679

358. The gold standard of treatment of femoral shaft fractures in adults is:A. Closed reduction and hip spica applicationB. Open reduction and reamed intramedullary nailingC. Closed locked antegrade intramedullary nailingD. Plating Answer: C; Schwartz 8th ed p.1688

359. Initial treatment of Talipesequinovarus (clubfoot) is:

Page 56: Surgery- board exam questions

A. Serial Casting immediately after birth/time of diagnosisB. Immediate operative treatmentC. Dennis Brown SplintD. Pavlik harness Answer: A; Schwartz 8th ed p.1718

360. A unique feature of immature bone is the capacity to undergo plastic deformation without breaking and to sustain an incomplete fracture also called:

A. Nighstick fractureB. Monteggia fractureC. Galleazzi fractureD. Greenstick fracture Answer: D, Schwartz 8th ed p.1698

361. Inflammation of the tendons in the first dorsal compartment containing the Abductor pollicis brevis and extensor pollicis brevis tendons is called:

A. Carpal Tunnel SyndromeB. Trigger fingerC. De Quervain’s TenosynovitisD. Lateral Epicondylitis Answer: C; Schwartz 8th ed p.1763

11.In doing an otoscopic examination of an adult , the ear is pulled a. upward and backwardsb. downward and anteriorlyc. upward and forwardd. downward and posteriorly

ans : a page 5 Boise funadamental of otolaryngology

12.This is a clinical hearing test using tunning fork , that compares the bone conduction of the examinaer from that of the patient.

a. Rinne Testb. Weber Testc. Swabach Testd. All of the above

Ans. c page 9 Boise funadamental of otolaryngology

13. This nerve controls the muscle of facial expression which is easily observe during face and neck examination

a. facial nerve ( CN VII)b. vagus nerve ( CN X)c. trigeminal nerve ( CN V)d. oculomotor nerve ( CN III)

Ans. A page 21 Boise funadamental of otolaryngology

14.The nasolacrimal duct empties into the:a. anterior ethmoidsb. inferior meatusc. lacrimal glandd. sphenoethmoidal recess

Ans. B page 179 Boise funadamental of otolaryngology

15. A 3 year old boy was brought to the ER due to recurrent epistaxis. On flexible nasal endoscopy, there was a bulging mass at the nasopharyngeal area. What is the \ possible diagnosis of the patient

a. Juvenile angiofibromab. Inverted papilloma

Page 57: Surgery- board exam questions

c. Nasal polypd. Fibrous dysplasia

Ans. A page 245 Boise funadamental of otolaryngology

16.This is a autosomal recessive trait characterized by situs invesus, bronchiectasis, sinusitis:

a. cystic fibrosisb. kartagener syndromec. Albrights syndromed. Pierre robin syndrome

Ans. B page 269 Boise funadamental of otolaryngology

17. This is a cellulitis or phlegmonous inflammation of the superior compartment of the suprahyoid space . The most common cause is odontogenic in origin

a. Vincents Anginab. Ludwigs anginac. Trench mouthd. Retropharyngeal abscess

Ans. B page 359 Boise funadamental of otolaryngology

19.the cricothyroid muscle is innervated by :a. recurrent laryngeal mnerveb. external br. of superior laryngeal nervec. internal branch of superior laryngeal nerved. vagus nerve

Ans B page 387 Boise funadamental of otolaryngology

20.The most common benign gland tumor of children of parotid gland is:a. lymphangiomab. hemangiomac. pleomorphic adenomad. mucoepidermoid CA

ANS B page 325 Boise funadamental of otolaryngology

21.A 22 year old construction worker comes for consult because an unknown liquid got into his eyes. The very first thing to do is:a) instill antisepticsb) do copious water irrigationc) assess visiond) put topical antibiotics

23. A 65 year old filipino male comes in for bilateral insidious painless progressive visual deterioration. The most likely diagnosis would be:a) cataractsb) age related macular degenerationc) central retinal artery occlusiond) congestive angle closure glaucoma

24.A 55 year old male known diabetic with moderately severe non proliferative retinopathy comes in for 6 months blurring of vision not corrected with pinhole. The most likely diagnosis is:a) macular edemab) traction retinal detachmentc) incipient cataractd) vitreous hemorrhage

Page 58: Surgery- board exam questions

25.A 41 year old female complains of slightly congested eyes with headache after prolonged near work, relieved by rest. She may needa) concave lensesb) convex lensesc) intraocular lensesd) binocular lenses

26.A patient complains of poor vision at distance, good vision at near. She may needa) convex lensesb) cylindrical lensesc) concave lensesd) binocular lenses

30.A chronic granulomatous inflammation of the meibomian gland is a a) cystb) stye c) chalaziond) dermoid

1. Following Goodsall’s rule, a fistula in-ano with an external opening located 4 cm from the anal verge would have its internal opening located :

A. in the anterior midlineB. in the posterior midlineC. radially, in the same quadrantD. radially, in a separate quadrant

2. In patients with rectal cancer, Computed Tomography scanning best evaluates:

A. The presence or absence of hepatic metastases.B. The depth of rectal wall penetration.C. The presence of lymph node metastases.D. The configuration of the primary tumor

1.Ultrasonography incidentally revealed a single 1 cm gallstone in an asymptomatic 55 year old. Which of the following is the recommended treatment?

A. Observation and follow-upB. Laparoscopic cholecystectomyC. Open cholecystectomyD. Bile acid dissolution therapy

2. 70-90% of primary gastrinomas are located in the area defined by a triangle with points located at the junction of the cystic duct and common bile duct, the second and third portion of the duodenum, and the neck and body of the pancreas. This area is known as the Triangle of

A. CalotB. HesselbachC. PassaroD. Bermuda

1. Histologic examination of a thyroid gland tumor reveals psammoma bodies. This finding indicates that the lesion is:

Page 59: Surgery- board exam questions

A. Follicular CarcinomaB. Medullary CarcinomaC. Papillary CarcinomaD. Hurtle Cell Carcinoma

2. A 40 year old female was diagnosed with Papillary Thyroid Carcinoma. She presents with a hard thyroid

mass, cervical lymphadenopathies, and several coin lesions on chest x-ray. Clinical staging for this patient

would be:

A. Stage IB. Stage IIC. Stage IIIE. Stage IV

3. A 45 year old woman has a mobile lump in the upper outer quadrant of her right breast. An open

excisional biopsy reveals a 1.5 cm lobular carcinoma in situ with negative margins. With regards to further management, which of the following statements is true?

A. Careful clinical follow-up without further surgical intervention is sufficient at this time.

B. Mirror image biopsy of the contralateral breast is warranted.C. Total mastectomy, without axillary dissection, is indicated.D. Lumpectomy with radiation therapy is the recommended treatment.

1. During the initial assessment of the multiply injured patient, the main goal and objective of the Primary Survey Is to:

A. obtain a detailed history and complete physical examinationB. stabilize the patient and comfort the relativesC. detect and treat immediately life-threatening injuriesD. detect and treat immediately limb-threatening injuries

2. In performing maneuvers to ensure airway patency, it is most critical to:

A. provide 100% oxygenB. have intravenous lines runningC. stabilize the cervical spineD. have intubation set ready

1. Respiratory distress in cases of congenital diaphragmatic hernia is a result of

A. inability of the ipsilateral lung to expand due to the herniated abdominal visceraB. recurrent aspiration pneumonia due to concomitant gastroesophageal refluxC. splinting of the contralateral diaphragm from distention of the intestinal tractD. incomplete development of the lung resulting in inadequate area for gas

exchange

Answer # questions page

B 1 Patients with either external or middle ear lesions usually have: 46A. Conductive hearing lossB. Sensorineural hearing loss

Page 60: Surgery- board exam questions

C. Mixed hearing lossD. none of the aboveMPL: 0.75

D 2 Oral or systemic antibiotics are indicated in Otitis Externa when: 82A. One is not sure if the patient has Otitis Externa or Otitis MediaB. The patient complains of severe painC. Debris is seen within the external auditory canalD. The infection extends beyond the limits of the canalMPL: 0.75

B 3 The most common etiology of acute otitis media in all age group 99A. Staphylococcus sp.B. StreptococcusC. H. InfluenzaeD. Moraxella catarrhalisMPL : 1.00

B 5 Which layer is missing when a tympanic membrane perforation heals 91A. Epidermal layerB. Fibrous layer C. Mucosal layerD. NoneMPL: 0.75

D 6 The most common intracranial complication from supurative Otitis Media is: 118

A. epidural abscess B. subdural abscess C. brain abscessD. meningitisMPL: 0.75

C 10 Most accurate test in detecting deafness in neonates and in children 67A. Play audiometryB. Pure tone audiometryC. Auditory Brainstem Evoked ResponseD. Otoacoustic emissionMPL: 0.50

C 11 The most common complaint of patients with otomycosis: 82A. EaracheB. Tragal tenderness C. Itchy earD. Non mucoid ear dischargeMPL: 1.00

B 12 Sudden hearing loss with concurrent dizziness while scuba diving suggests: 94A. Acoustic neuroma B. Barotrauma C. Meniere’s diseaseD. Benign Positional Paroxysmal VertigoMPL: 0.75

A 14 Conditions predisposing to the development of otitis media 100A. Upper respiratory tract infectionB. Allergic rhinitisC. Breastfeeding D. Cleft palateMPL: 0.75

A 21 A 3 year old boy was refered to the ENT service due to edema of the eyelids. 236-264 Pertinent physical examination: normal visual acuity, (-) chemosis, (-) proptosis, full extraocular muscles, (+) profuse purulent nasal discharge. What is your assessment?

A. Periorbital cellulitisB. Orbital cellulitisC. Subperiosteal abscessD. Orbital abscess

Page 61: Surgery- board exam questions

MPL: 0.75

B 22 The most common cause of epistaxis in the elderly 236A. Trauma B. Atherosclerosis and hypertensionC. Foreign body D. Blood dyscrasiaMPL: 1.00

C 27 Which of the following best describe a patient with allergic rhinitis? 202A. Presence of purulent nasal discharge with obstructionB. b. Presence of watery nasal discharge with headacheC. c. Presence of watery nasal discharge with obstructionD. d. Presence of purulent discharge with headacheMPL: 1.00

A 28 Which of the following best describe vasomotor rhinitis 225A. A lady dentist with nasal obstruction throughout her pregnancyB. A jeepney driver with nasal obstruction and sneezingC. A medical intern with one sided nasal obstruction and headache D. A farmer with nasal obstruction and crusts in the nasal cavityMPL: 0.75

D 29 Rhinitis medicamentosa is due to overuse of 203A. Oral steroidsB. Oral decongestantsC. Local steroidsD. Local decongestantsMPL: 0.75

C 30 Which of the following best describe a patient with juvenile angiofibroma 245A. A 24 y/o female bank teller with epistaxis during office hours B. A 66 y/o male, smoker, with intranasal massC. A 14 y/o boy, with profuse epistaxisD. A 53 y/o hypertensive female with epistaxisMPL: 0.75

C 31 Which is the most critical aspect of management in odontogenic infection: 297A. Appropriate cidal antibioticB. Perform aggressive incision and drainageC. Remove the offending toothD. Work-up to identify possible mediastinal extensionMPL: 0.75

A 33 A 50-year old female presents clinically with globus hystericus. This is usually 392 associated with a/an:

A. A normal PE and barium swallowB. Lesion of the upper esophagusC. Abnormal direct laryngoscopyD. Structure of the upper esophagusMPL: 0.75

B 34 A 20-year old sailor has a persistent sorethroat and is found to have a pharyngitis 344 with enlarged tonsils and cervical adenophaty . Gram stain of pharyngeal secretions shows gram- negative diplococci. These findings support a diagnosis of:

A. Streptococcal pharyngitis B. Gonococcal pharyngitis C. Haemophilus influenzaD. Bacteroides pharyngitisMPL: 1.00

D 35 A 25y/o male consulted at the OPD due to dysphagia of 2 days duration. 346 PPE: (+) trismus, increased salivation, peritonsillar swelling (left) pushing uvula across midline. What is your working diagnosis?

A. Acute tonsillitis B. Acute pharyngitis C. Acute tonsillophayngitisD. Peritonsillar abscessMPL: 0.75

Page 62: Surgery- board exam questions

C 36 This lymphoid tissue may be completely removed because of the presence of a 350-351 capsule:

A. Guerlach’s tonsilsB. Pharyngeal tonsilC. Palatine tonsilsD. Lingual tonsilMPL: 0.75

C 37 A 50 year-old male had barium swallow with the report of aperistalsis, esophageal 471 dilatation, and failure of the lower esophageal spinchter to relax. Which of the following is your primary consideration?

A. PresbyesophagusB. Diffuse Esophageal SpasmC. AchalasiaD. SclerodermaMPL: 0.50

C 38 Tonsillectomy should be considered for all of the following except: 352A. Obstructive sleep apneaB. Recurrent peritonsillar abscessC. Asymptomatic tonsillar hyperplasiaD. Unilateral tonsillar hyperplasiaMPL: 0.75

A 39 Complication of a retropharyngeal abscess may involve: 362A. MediastinitisB. Hypoglossal paralysis C. Horner’s syndromeD. ThyroiditisMPL: 0.25

A 40 The region of the pharynx that extends from the base of the skull to the level of 274hard palate:A. Epipharynx B. Mesopharynx C. HypopharynxD. LaryngopharynxMPL: 0.50

C 41 Majority of FB coins will be trapped in the: 475A. Gastroesophageal constrictionB. Aortic and bronchial constriction in the esophagusC. Cricopharyngeal constrictionD. TonsilsMPL: 0.75

A 1 1/2 month-old baby girl was brought to your clinic because of stridor. There was no associated cough. The patient was delivered to a G8P7 40 year old laundrywoman from payatas via NSD. Birthweight was 6.5lbs with APGAR score of 9 and 10. She was sent home after 2 days with no perinatal complications. She has a good suck with normal swallowing, good cry and weight gain. Stridor is slightly relieved by putting her in a supine position. T=37.2 C ; RR= 40/min.

D 43 Your initial impression is: 393-394A. Congenital subglottic stenosisB. Congenital laryngeal webC. LaryngocoeleD. LaryngomalaciaMPL: 0.75

D 44 The best thing to do in this patient for your initial evaluation is/are: 393A. Request for a plain chest x-rayB. TracheostomyC. Request for CT scan of the neckD. Direct laryngoscopyMPL: 0.75

Page 63: Surgery- board exam questions

A 6 year-old boy was brought in to the clinic because of cough associated with moderate-grade fever and sorethroat characterized by painful swallowing few days PTC. The patient was sent home with antibiotics, cough syrup and antipyretics. The following day, patient was brought to the ER because of stridor associated with cyanosis. Patient was relieved by sitting up with mouth open and chin forward. He was restless and cyanotic with flaring of alae nasi, substernal and intercostal retractions. T=40C ; RR=36/min.

B 45 Your initial impression is: 399A. Drug allergyB. Acute epiglottitisC. Acute subglottic laryngitisD. Foreign body in laryngotracheobronchial treeMPL: 1.00

D 46 The best thing to do in this patient as your initial evaluation and management is/are: 399A. Chest X-ray B. Tracheostomy C. LaryngoscopyD. AntibioticsMPL: 0.75

A 62 year old farmer from Iriga City came in because of dyspnea. His condition started 10 months PTC as hoarseness associated with hemoptysis. One month PTC, he developed persistent dyspnea accompanied by a lump on the right side of the neck. Patient is a chronic alcoholic and smoker. On PE, patient is obese, afebrile, and in respiratory distress.

D 47 Your initial impression is: 461A. Chronic non-specific laryngitis B. Pedunculated vocal cord polyp C. Laryngeal papillomaD. Laryngeal CAMPL: 0.75

C 48 The patient requires: 461A. Chest X-rayB. CT scan of the neck C. Direct laryngoscopy and biopsyD. Neck node excision biopsyMPL: 0.75

A 3 year-old girl was taken to the ER because of stridor and cyanosis. Few hours PTC, patient was playing in the sala while her daddy was eating peanut while watching world meeting of families. The patient was restless with audible slap and palpable thud.

B 49 Your initial impression is: 484A. Bronchial foreign body B. Tracheal foreign body C. TracheomalaciaD. Esophageal foreign bodyMPL: 0.50

A 50 Your initial diagnostic procedure is: 484A. Chest X-ray B. CT scan of the chestC. CT scan of the neckD. Direct laryngoscopyMPL: 0.25

A 51 The only bony structure in the laryngeal skeleton is the 384-385A. hyoid B. thyoid d. C. cricoidD. epiglottisMPL: 1.00

Page 64: Surgery- board exam questions

C 53 The best way to examine the larynx in the OPD is by 392A. direct laryngoscopyB. palpationC. indirect laryngoscopyD. lateral neck X-raysMPL: 1.00

D 54 The most common congenital laryngeal anomaly is 384A. subglottic stenosisB. vocal fold paralysis C. laryngeal hemagiomaD. laryngomalaciaMPL: 1.00

A 55 The primary structure preventing laryngeal aspiration is: 390A. True vocal cords B. False vocal cordsC. EpiglottisD. ValeculaMPL: 0.75

A 62 Symptoms of mandibular fracture include 532A. malocclusion B. epistaxis C. a flat faceD. lateral neck numbness

MPL: 1.00

C 63 X-ray evidence of unilateral haziness of the maxillary sinus in a trauma case 528 indicates

A. foreign body impaction in the sinusB. total collapse of the sinus wallsC. blood in the sinusD. sinusitisMPL: 0.75

B 64 The treatment of choice for depressed zygomatic arch fracture is 535A. Caldwell Luc OperationB. Gillie's OperationC. Forced Duction OperationD. Titanium plate and screw fixation.

MPL: 0.25

B 65 Procedure of choice for neglected nasal bone fracture 531A. ORIF B. Septorhinoplasty C. Closed reductionD. SeptoplastyMPL: 0.75

B 66 Tear drop” sign indicative of blow-out fracture is seen best on: 535A. AP viewB. Water’s view C. Vertex viewD. Transorbital viewMPL: 0.50

C 67 A simple depressed fracture of the nose is best treated by: 531A. Simple septorhinoplastyB. Open reductionC. Closed reductionD. Titanium plating

B 68 The second most common fracture in the face is: 532A. nasal bone fractureB. Mandibular fractureC. c. Alveolar ridge fracture

Page 65: Surgery- board exam questions

D. d. Zygomatic fractureMPL: 1.00

B 69 The Le Fort II fracture is also known as 537A. Guerin fractureB. Pyramidal fractureC. Craniofacial dysjunctionD. Rhomboid fractureMPL: 1.00

A 70 The Le Fort I fracture is also known as 537A. Guerin fractureB. Pyramidal fractureC. Craniofacial dysjunctionD. Rhomboid fractureMPL: 1.00

C 72 Repair of a cleft palate is best done 288A. soon after birthB. before school age C. before speech developsD. before dating age (adolescent).MPL: 1.00

C 73 The operation for prominent ear lobe deformity is called: 96A. tympanoplastyB. myringotomyC. otoplastyD. auditory meatoplastyMPL: 1.00

C 76 Scar tissue formation may be delayed by using 517A. antibioticsB. lidocaine injection C. steroidsD. epinephrineMPL: 0.75

B 77 A free flap derives its blood supply from: 514A. A neighboring artery in the recipient siteB. The capillaries in the recipient siteC. The arterial supply in the donor siteD. The capillaries in the donor siteMPL: 0.75

D 78 In wound healing, during the stage of injury, which of the following occurs: 505A. PMN leukocytes predominate the population of inflammatory cells in woundB. wound is filled with extracellular matrixC. fibroblasts fill up the woundD. formation of hemostatic plug composed of fibrin and plateletMPL: 0.75

B 79 The most usual donor site for split thickness skin graft 512A. neckB. thighC. abdomenD. post-auricular areaMPL: 0.50

B 80 Which of the following grafts has a better chance of “take”(survival of the graft) ? 516A. Full thickness skin graftB. Split thickness skin graftC. Free bone graftD. Autogenous graftMPL: 0.75

B 81 A 54-year-old patient presented with a midline mass that moves with deglutition. 439-440 Your primary diagnostic impression would be, that maybe it’s a/an:

Page 66: Surgery- board exam questions

A. Thyroglossal duct cyst that manifested late in life.B. Thyroid nodule, probably benignC. Thyroid malignancyD. Inflammatory noduleMPL: 0.75

A 82 The most logical question to ask when probing whether the mass is congenital, 431 would be:

A. What is the age of the patient?B. Is the mass growing rapidly?C. Is the mass cystic or solid?D. Is there any source of infection?MPL: 1.00

C 83 A 30 year old female comes in with fever, pains in the right pre-auricular area. On 319 examination, you see a swollen, tender, erythematous right parotid area. The orifice of the stensen’s duct is also swollen with minimal purulent material coming out. What is the diagnosis?

A. Sialolithiasis B. Chronic sialadenitisC. Acute bacterial sialadenitisD. MumpsMPL: 1.00

D 84 A 40 year old male presents with a 3 X 3 cm right pre-auricular mass. FNAB 327showed pleomorphic adenoma. Which of the following statements is incorrect?A. It is the most common benign neoplasm in the parotid glandB. The tumor has a slow growth patternC. Treatment of choice is superficial parotidectomy with facial nerve preservationD. Recurrence is not a problem in such a case. MPL: 0.50

D 85 A 20 y/o male came to the clinic with a mass at the left submandibular area. 319Primary consideration as to the nature of the mass would probably be:A. Neoplastic B. Congenital C. CysticD. InflammatoryMPL: 1.00

A 60 y/o female teacher from Iriga City came in for a right pre-auricular mass of 2 months duration. Initially the patient palpated a tender 1 X 0.5 cm mass which progressively enlarged to its present size. On P.E there is 3 X 4 X 2 cm tender, movable, firm to stony hard, nodular mass on the right pre-auricular area. The overlying skin is not indurated and moves freely over the mass. No cervical lymphadenopathy was noted.

D 86 The finding in the above patient which supports the possibility of a parotid gland 431 malignancy:

A. The age of the patientB. The absence of cervical lymphadenopathyC. The presence of pain and duration of the massD. The age of the patient and duration of the massMPL: 0.75

B 91 Surgical treatment of choice for thyroglossal duct cyst is 434A. Marsupialization procedureB. Excision of cyst tract and portion of hyoid bone C. Excision of cyst and tractD. Removal of hyoid boneMPL: 0.50

B 93 The treatment of choice in malignant lymphoma is/are: 437A. SurgeryB. Radiotherapy and/or chemotherapyC. ChemotherapyD. Surgery and/or chemotherapyMPL: 0.50

C 94 The earliest sign of carcinoma of the larynx is 444A. Sore throat

Page 67: Surgery- board exam questions

B. Dysphagia C. HoarsenessD. DyspneaMPL: 0.75

C 95 The most common histologic type of carcinoma in the head and neck area is 445A. AdenocarcinomaB. Basal cell carcinoma C. Squamous cell carcinomaD. SarcomaMPL: 0.75

B 96 The most commonly benign salivary gland tumor in children: 325A. PapillomaB. HemangiomaC. Pleomorphic AdenomaD. Mucoepidermoid carcinoma

MPL: 0.50

B 97 Knowledge of the lymphatic drainage is important because it will: 448A. Go to the primary lesionB. Give an idea where the primary lesion isC. Be evident in the histopathD. Distribute lymph as a defense mechanismMPL: 1.00

C 98 A neoplasm that occurs primarily in the parotid gland, more commonly seen in 327males in the older age group and is also known as papillary cystadenoma lymphomatosumA. OncocytomaB. Pleomorphic adenomaC. Warthin’s tumorD. Hemangioma MPL: 0.75

C 99 Sudden facial paralysis associated with vesicles in the external audiotory 83canal and pinna:A. Melkersson’s syndrome B. Bell’s palsy C. Ramsay hunt syndromeD. SchwannomaMPL: 0.75

C 100 The opening of the parotid duct is at level: 317A. 1st upper molar B. 1st lower molar C. 2nd upper molarD. 2nd lower molarMPL: 1.00

1. In which of the following conditions is the cornea most opacified? MPL = 0.25a. Maculab. Leukomac. Nebulad. Corneal abrasion

2. A condition that gives painless blurring of vision is: MPL = 1.0a. Central retinal vein occlusionb. Orbital cellulitisc. Optic neuritisd. scleritis

3. The most important factor for developing diabetic retinopathy is: MPL = 0.25a. Duration of the diabetesb. Poor metabolic control of diabetes

Page 68: Surgery- board exam questions

c. Concomitant renal diseased. Concomitant systemic hypertension

4. In hyperopia, the axial length is: MPL = 1.0a. Too shortb. Too longc. Just rightd. Of no significance

5. The leading cause of avoidable and reversible blindness worldwide is/are … MPL = 0.5a. …Cataracts.b. …Corneal leukomasc. …Trauma.d. …Glaucoma

6. The most useful chart attractive to preschool illiterate children: MPL = 0.5a. Snellen Chartb. Illiterate ‘E’ Chartc. Allen Chartd. ETDRS Chart

7. One advantage of Direct Ophthalmoscopy over Indirect Ophthalmoscopy is: MPL = 1.0a. Stereoscopyb. Magnificationc. Light Intensityd. Wider Field of Vision

8. Flouresecin dye when viewed using a cobalt blue light will appear as: MPL = 0.5a. Luminous orangeb. Luminous redc. Luminous blued. Luminous green

9. Blind spot on a normal visual field is also known as: MPL = 1.0a. Scotomab. Epiphorac. Diplopiad. Asthenopia

10.Most accidental eye injuries can be prevented by… MPL = 0.5a. …a good pair of protective gogglesb. … a good surgical maskc. ... common sensed. all of the above

11.The most important consideration in Testing Near Acuity: MPL = 0.25a. Ageb. Testing Distancec. Literacyd. Stereoscopic Vision

12.Determination of Visual Field Extent is best demonstrated by: MPL = 0.25a. Octopus Perimetryb. Automated Keratometryc. Amsler Grid Testingd. Confrontation Fields Test

Page 69: Surgery- board exam questions

13. In performing Direct Ophthalmoscopy, the first element that must be observed: MPL = 0.5

a. Media Clarityb. Red reflexc. Macular changesd. Cup / Disc features

14. In the three-part step-wise sequence in external eye examination, one procedure that is not usually included: MPL = 1.0

a. Inspectionb. Auscultationc. Palpationd. Percussion

15.The patient was not able to see hand movement. What is the next step to check his visual acuity?MPL= 0.5

a. Check light perception*b. Have the patient walk closer to the Snellen chart and ask if he can see the

biggest letterc. Do counting fingersd. Check color perception test

16.A patient is only able to read the first line of the Jaeger chart. His near visual acuity is recorded as…MPL=0.3

a. J16b. J16 -2 lines*c. J16 +2 linesd. J16 +2

17. In assessing facial sensation, the three branches of the cranial nerve V that is being tested are…MPL=0.3

a. Supraorbital n., infraorbital n., intraorbital n.b. Facial n., abducens n., trochlear n.c. Frontal n., zygomatic n., lacrimal n.d. Ophthalmic n., maxillary n., mandibular n.*

18.Which of the following is/are true of the corneal blink reflex? MPL=0.3a. Test is done by touching the eyelashes lightly with a wisp of cottonb. The afferent arm of the reflex is the trigeminal nerve*c. The efferent arm of the reflex is the optic nerved. Visual startle reflex should be stimulated

19.Which of the following is a term used to describe normal binocular eye movement in the same direction? MPL=0.5

a. Ductionsb. Versions*c. Vergenced. Convergence

20.Giant papillary reaction is common in… MPL = 1.0a. Contact lens overuseb. Preseptal cellulitisc. Orbital cellulitisd. Chalazion

21.Ophthalmia Neonatorum is usually a bacterial cause of conjunctivitis: MPL =0.3a. True

Page 70: Surgery- board exam questions

b. Falsec. Only rarely of bacterial caused. Real causative organism is unknown

22.The spread of epidemic viral conjunctivitis is best curtailed by: MPL =0.25a. Putting prophylactic medication on the eyesb. Avoiding crowded placesc. Avoid touching one’s own eyesd. Frequent hand washing

23.Which is not an anatomic locale description of the conjunctiva? MPL =0.5a. Tarsalb. Fornixc. Limbald. Bulbar

24.Adenoviral conjunctivitis may cause: MPL =0.3a. a true membrane on the tarsal conjunctivab. a pseudomembranec. never forms any membraned. may develop keratinized conjunctiva

25.Which of the following statements regarding the use of steroid eye drops for red eyes is correct? MPL =0.3

a. Patients may self medicate with steroids when they have red eyes.b. Steroids may be used so long as the dosage is less than 4 times a day.c. The possible complications of steroid use include Glaucoma AND Cataract

formationd. Ocular complications of steroid eye drops are independent of dosage and

duration of use

26.The medication specifically used against adenovirus conjunctivitis currently is: MPL =0.3

a. Steroid antibiotic combination eye drops.b. 4th generation flouroquinolone eye drops (moxifloxacin)c. Interferon eye dropsd. There is no specific medication against adenovirus

27.The definitive management of gonococcal conjunctivitis is: MPL =0.3a. Tobramycin eye drops every hour to affected eye for 10 days.b. Ceftriaxone 1gram intramuscular injection once only.c. Gentamycin intravenous, 200mg q 6hrs for one week.d. Oral amoxicillin, 500mg t.i.d. for one week

28.Which of the following conjunctivitides is almost always bilateral? MPL =0.5a. Viralb. Bacterialc. Toxicd. Allergic

29.Fusion of the bulbar and tarsal conjunctiva as a result of prolonged inflammation and scarring is termed: MPL =0.3

a. Ankyloblepharon.b. Symblepharonc. Eryblepharond. Blepharitis

Page 71: Surgery- board exam questions

30.Painful blurring of vision can be due to: MPL = 1.0a. Hypertensive Retinopathyb. Acute angle closure glaucomac. Vitreous hemorrhaged. Mature Cataract

31.Glaucoma is characterized by: MPL = 0.25a. Blurred disc marginb. Exudatesc. Conjunctival discharged. Damage to optic nerve head

32.Secondary glaucoma may occur in: MPL = 0.25 a. Hyphemab. Acute Bacterial conjunctivitisc. Central retinal vein occlusiond. Retinal detachment

33.The hallmark of Grade IV hypertensive retinopathy associated with malignant hypertension is: MPL = 0.25

a. silver-wiring of arteriolesb. macular ‘star’ (hard exudates)c. optic disc edemad. cotton-wool spots

34. The systemic condition that presents with the most complications in the posterior pole of the eye: MPL = 0.25

a. Tuberculosisb. Diabetes Mellitusc. Allergyd. Blood dyscrasias

35. In testing for strabismus, the corneal light reflex did not fall at the same relative position on each eye instead it was located at the temporal borders of the pupils, the patient has: MPL = 0.5

a. Orthophoriab. Esotropia c. Exotropiad. Hypertropia

36. In cover testing, the uncovered eye moves inward to fixate, the patient has: MPL = 0.5a. Orthophoriab. Esotropiac. Exotropia d. Hypertropia

37.A 5 year old child was brought to an ophthalmologist because his family had noticed a strange, white reflection that appeared in one of his eyes in photographs while the rest have red eyes due to the flash. Further examination disclosed Retinoblastoma. This white pupil is known as: MPL=0.5

a. pseudohypopyonb. leucocoriac. Hyphemad. Red eyee. Strabismus

Page 72: Surgery- board exam questions

38.Computed Tomogram of the above child showed a unilateral tumor filling the left eye. Nasal third of the tumor is calcified. Impression of Retinoblastoma is confirmed with the following histologic features: MPL=0.5

a. Islands of blue cells in a sea of pink necrosisb. Cuboidal cells circled around a central luman: Flexner-Wintersteiner rosettesc. Flower like grouping of tumor cells which look like photoreceptors: Fleurettesd. All of the abovee. B and c only

39.A 23/M consulted the OPD because anterior eyelid crusting of 3 days duration. The resident requested for a gram-stain which revealed Gram-positive cocci in clusters. Which statement is consistent with the impression of Staphylococcal blepharitis? MPL = 0.5

a. Coagulase production specifies etiologic agent to be Staphylococcus aureus

b. Catalase negative reaction identifies normal flora Staphylococcus epidermidisc. All staphylococcus species are catalase negatived. B-lactamase production increases susceptibility to penicillins

40.Which of the following viruses is transmissible even after medical instrumentation is cleaned with alcohol? MPL = 0.5

a. Herpes simplex virusb. Adenovirusc. Human immunodeficiency virusd. Epstein-Barr virus

41.A newborn was admitted to the NICU because of mucopurulent eye discharge at birth. The mother had a documented chlamydial genito-urinary tract infection. What bacterial features are consistent with the neonatal ocular finding? MPL = 0.5

a. Elementary bodies stain blue with Giemsa in contrast to the purple of the host cell cytoplasm

b. Chlamydia trachomatis strains synthesize folates thus are resistant to sulfonamides

c. The outer cell wall resembles the cell-wall of gram-negative bacteriad. Chlamydiae are obligate intracellular parasites that need to exist within host cells

to synthesize ATP

42.A 50/M farmer consulted the OPD because of right corneal opacity of 3 weeks duration. He claimed his right eye was hit by a palay strand while harvesting. Clinical history alone leads to a strong suspicion of fungal keratitis. Which of the following statements is a recommended diagnostic management of fungal keratitis? MPL = 0.5

a. Fungi can easily be visualized using the routine hematoxylin and eosin (H&E) preparation

b. The cell wall polysaccharides are uniquely stained by periodic acid-Schiff (PAS)

c. The gold standard for morphologic classification is blood agar at room temperature

d. Corneal scrapings are best collected at the center of the main lesion rather than the small satellite lesions

43.A 43/F consulted because of a sessile mass at the limbus. The excision biopsy revealed squamous papilloma. Which of the following is true about the association of the Human Papilloma Virus (HPV) with squamous papilloma? MPL = 0.5

a. HPV DNA induces cellular necrosis leading to a secondary papilloma growthb. HPV subtypes 6 and 11 have been identified in hypertrophic papillomasc. HPV DNA induces cellular proliferation and can lead to malignancy

Page 73: Surgery- board exam questions

d. HPV RNA integrates with epithelial genome leading to squamous cell carcinoma

44.A 30/M consulted the OPD because of foreign body sensation with mucopurulent discharge of 6 days duration. A detailed slit-lamp biomicroscopic examination demonstrated clinical features of trachoma. Which are features of an adult trachoma compared to a neonatal chlamydial conjunctivitis is TRUE? MPL = 0.5

a. The amount of mucopurulent discharge is greaterb. Membranes develop almost immediatelyc. The percentage of intracytoplasmic inclusions is greaterd. Follicular response is greater

45.Condition in which there is discoloration of the eyelashes MPL = 0.5a. Madarosisb. Proptosisc. Trichiasisd. Poliosis

46.The condition in which an extra row of lashes exists is MPL =0.5a. Trichiasisb. Distichiasisc. Entropiond. Ectropion

47.A white pupillary reflex is called: MPL = 1.0 a. leukomab. glaucomac. leukocoriad. anisocoria

48.An infant born less than 31 weeks, weighing less than 1500g (3 lb 5 oz) plus exposure to supplemental oxygen, one should screen the eyes for: MPL = 1.0a. Persistent Hyperplastic Primary Vitreous (PHPV)b. congenital cataractc. a retinal astrocytomad. Retinopathy of Prematurity (ROP)

49.This is the second most common mode of presentation in Retinoblastoma which account for about 20 % of cases: MPL =0.5a. leukocoriab. secondary glaucomac. proptosisd. strabismus

50.The following are treatment options for Retinoblastoma except MPL = 1.0a. Cryotherapyb. Radio Therapyc. Enucleationd. Evisceration

51.The following are correct in visual acuity testing in a child EXCEPT MPL=0.5a. At 6 months, test for ocular fixation and ocular movementb. At 4 years old, test with tumbling “E” chart picture chartc. At 8 years old, test with an Amsler grid chart d. At 16 years old, test with a Snellen chart

Page 74: Surgery- board exam questions

52.Leukocoria caused by the presence of red cells in the vitreous secondary to birth trauma MPL=0.5a. Retinopathy of prematurityb. Persistent Hyperplastic Primary Vitreousc. Vitreous Hemorrhage d. Posterior Uveitis

53.A biconves, avascular, colorless structure that focuses light rays to the retina, an opacity of which will cause leukocoria MPL=0.5a. Lens b. Vitreousc. Retinad. Optic Nerve

54.Ophthalmological examination that would identify presence of an intraocular mass MPL=0.5a. Visual acuity testingb. Indirect Ophthalmoscopy c. Pupillary examinationd. Sit lamp examination

55.The most common presenting sign of retinoblastoma is MPL=1.0a. Red eyeb. Visual lossc. White pupil d. Strabismus

56.Due to the inheritance pattern of retinoblastoma, a vital part of treatment would incude: MPL=0.5a. Psychological counselingb. Genetic counseling c. Psychotherapyd. Rehabilitation counseling

57.A surgical procedure involving removal of intraocular contents: MPL=0.5a. enucleationb. eviscerationc. exenterationa. retrobulbar alcohol

58.Management of CRAO includes MPL =0.5a. “Brown bagging”b. Alternate compression and decompression of the globec. Oxygen inhalationd. A and B onlye. B and C only

59.Characteristic findings in Central Retinal Artery Occlusion MPL = 0.25a. Cotton wool spotsb. Cherry red spot in the maculac. Vitreous floatersd. Papilledema

60.The TRUE statement regarding chemical burns MPL = 1.0a. Acid burns cause more extensive damage than alkali burns

Page 75: Surgery- board exam questions

b. The first thing to do in cases of chemical burns is to check the visual acuityc. Neutralization should be achieved, so that an alkali burn can be treated by

instilling an acidic substance and vice versad. In copious irrigation of the eye, the fluid used need not be sterile, provided

the chemical is diluted properly

61.EOM movement of one eye is called MPL =0.5a. Ductions b. Versionsc. Nystagmusd. Diplopia

62.Absence of venous pulsations on funduscopy MPL =0.3a. May indicate increase intracranial pressureb. May be normalc. Is a sign of glaucomad. A and b

63.To examine the optic nerve binocularly, use MPL = 0.3a. Indirect ophthalmoscopeb. Non contact fundus lensc. Intraocular lensd. A and B

64.What ancillary procedures should you do if suspecting optic nerve toxicity? MPL = 0.25a. Visual field examb. Visual evoked potentialc. Color testd. All of the above

65.One of the most common reasons for Neuroophtha referral … MPL = 1.0a. …unexplained vision lossb. …cataractsc. …retinal disordersd. none of the above

66.Which of the following is true regarding viewing the fundus… MPL = 1.0a. Indirect ophthalmoscopes provides a monocular view of the fundusb. Direct ophthalmoscopes provides a Binocular view of the fundusc. Direct ophthalmoscopy provides greater detail of the fundus d. none of the above

67.What is the most important adverse effect of local anesthetic overdosage? MPL = 0.25a. bronchoconstrictionb. renal failurec. convulsiond. skin rashes

68.Cortocosteroid stops the inflammatory process by inhibiting prostaglandin synthesis through the following mechanism: MPL = 1.0a. blockage of the enzyme cyclooxygenaseb. blockage of the enzyme phospholipase Ac. blockage of GABA pain receptorsd. regulation of endorphin levels in the blood

Page 76: Surgery- board exam questions

69.Which of the following statements regarding papilledema is not true? MPL =0.3a. Loss of venous pulsation is always presentb. Symptoms accompanying papilledema may include visual loss and diplopiac. Nerve fiber layer hemorrhages around the disc are often presentd. Blurring of the peripapillary vessels

70.Most common visual field finding in papilledema MPL = 0.3a. Central scotomab. Cecocentral scotomac. Altitudinal defectd. Enlargement of the blind spot

71.Typically the onset of visual loss in demyelinating optic neuritis is noted MPL = 0.25a. Upon arising in the morningb. Mid-dayc. At the end of the dayd. There is no temporal pattern for visual loss

72.One of the following findings is not a characteristic of true disc edema MPL = 0.25a. capillary dilationb. abnormal branching of vessels around the discc. disc hemorrhagesd. blurring of disc vessels along the margin

73.Most common cause of optic disc swelling in children MPL = 0.3a. Infectious optic neuritisb. Demyelinating optic neuritisc. Leber optic neuropathyd. Non arteritic anterior ischemic optic neuropathy

74.One of the following is not a characteristic finding of optic neuritis in children MPL = 0.25

a. Often present simultaneously as a bilateral conditionb. More often anterior in location with optic disc edema seen on examc. Occurs 1-2weeks after a known or presumed viral infectiond. Always resistant to steroids

75.Most common condition associated with nonarteritic anterior ischemic optic neuropathy MPL = 0.5

a. Diabetes mellitusb. Carotid artery diseasec. Coagulopathies d. Hypertension

76.Which of the following is the most common optic neuropathy in patients over the age of 50? MPL = 0.3

a. Compressiveb. Demyelinatingc. Traumaticd. Nonarteritic anterior ischemic optic neuropathy

77.10. An obese 26 year old woman presents with headaches and transient visual obscurations. Bilateral papilledema is documented. The next step should be which of the following MPL = 0.3

a. advised weight reduction

Page 77: Surgery- board exam questions

b. do lumbar tapc. Start on acetazolamided. Do CT or MRI of the brain

78.Exophthalmos is a term used specifically in which of the following conditions? MPL = 1.0

a. Thyroid diseaseb. Anemiac. Keratoconusd. Myopia

1. The most commonly observed Cumulative Trauma disorder isA. De QuervainsB. Trigger fingerC. Carpal tunnel syndromeD. Tennis elbow

2. The ff. is an indirect cost of injury among workersA. medical CostB. Lost Time WagesC. Training replacementsD. Compensation Premiums

3.The ff. is an indication for surgical release of CTSA. Severe causalgia B. Weak Abductor Pollicis brevisC. Atrophy of hypothenar eminenceD. edema on the wrist

4. The ff. is a feature of Impingement Syndrome A. Limited Passive ROM of the shoulderB. Pain active on shoulder flexionC. Limited active abduction of the shoulderD. Positive Spurling sign

5. In Reflex sympathetic dystrophy the ff. is trueA. Pathology originates at the fingersB. Autonomic changesC. Complete shoulder ROMD. Splinting the shoulder helps a lot

6. The most common lump on the hand isA. Trigger fingerB. Ganglion cystC. Tendenitis D. Dupuytrens contracture

7. The Following can lead to Frozen Shoulder EXCEPTA. Rotator cuff TearB. Bicipital tendenitisC. Carpal Tunnel SyndromeD. Trigger finger

8. Avascular necrosis of the LunateA. PannersB. KeinbocksC. Dupuytrens

Page 78: Surgery- board exam questions

D. Osgood

9. A worker comes with a lump on the dorsal elbow, it is movable, soft with tenderness. He has a habit on putting his weight at the elbow , he has A. Golfer’s elbowB. Tennis elbowC. TendinitisD. Student’s elbow

10. The best thing to do in acute tendenitis isA. To stretch it to increase tensile strengthB. To rest it in a splint C. To move it to prevent contractureD. To apply hot moist pack

11. The grip strength of a dominant hand is usually stronger than the non dominant hand by A. 10 lb. C. 30 lbsB. 20 lb. D. 40 lbs

12. A functional non organic sensation deficit is evident if there isA. vibratory deficit in the upper armB. sensory deficit in the whole leg C. numbness of the thumbD. numbness on the right side of the face

13. To prevent overvaluation and subjection to potential risk of tests. The following will suggest a non organic findings in back painA. Pain on straight leg raising at 30 degreesB. light pinch tenderness of skin on vast area of the backC. Sensory deficit on the lateral thighD. weak extensor hallucis longus

14. The most common type of muscular dystrophy isA. Limb Girdle B. DuschenneC. Facioscapulo-humeralD. Beckers

15. Cramps is the usual complain ofA. DuschenneB. MyotonicC. SMAD. Limb Girdle

16. Spinal dysraphism can be detected in utero after 18 weeks by taking the A. Amniotic C -reactive proteinB. Amniotic Alpha feto proteinC. Amniotic phosphofructokinaseD. Amniotic amylase

17. A positive Gowers sign is A. Pathognomonic in muscular dystrophyB. Present in pelvic and proximal leg weaknessC. seen in hip flexion contracture D. a feature of McArdle’s disease

Page 79: Surgery- board exam questions

18. Ischemic Compression Test is for A. Duschenne MDB. MyotoniaC. SMAD. McArdle’s Disease

19. A dive bomber sound on EMG is seen inA. Beckers MDB. MyotoniaC. Spinal Dysraphism D. SMA

20. Fasciculation’s of the tongue and limb muscles is frequently seen in A. Beckers MD C. Spinal DysraphismB. Myotonia D. SMA

21. Most of children with this condition cannot reach adulthood exceptA. DuschenneB. Fascioscapulohumeral dystrophyC. Werdnig Hoffman (Acute)D. Pompe’s Disease

22. The most common cause of death of children with myopathies isA. Renal Problem C. Pulmonary problem B. Weakness D. Myocardial infarction

23. This reflex is integrated in the spinal cordA. Asymmetric tonic neck reflexB. Optical rightingC. Flexor withdrawalD. Landau reflex

24. This reflex develops mouth opening, helps find the breast and develops various tongue position

A. Sucking reflexB. Rooting reflexC. Automatic head turningD. Demand feeding reflex

25. Asymmetric Tonic Neck reflexA. Enables each side of the body to be used separatelyB. Coordinates with tonic labyrinthine reflex for turning the bodyC. Integrated in the cortexD. Appears at six month of age

26. Patient with Spina bifida should be examined forA. Presence of strabismusB. Presence of Mongolian spotC. Presence of hip dislocationD. Presence of hormonal problem

27. . This instrument is used to measure range of motion of joints:A. tape measure C. tachometerB. goniometer D. planometer

Page 80: Surgery- board exam questions

28. . Which of the following is considered as a deep heating modality:A. heating pads C. ultrasoundB. whirlpool baths D. paraffin baths

29. On stimulation the baby extend and abduct the limbs followed by flexion and adductionA. If this is a normal baby he is less than four monthsB. This is a reflex integrated in the midbrainC. This reflex will not interfere in balance and rolling overD. This is a normal reaction in baby of all ages

30. . Stroke rehabilitation may be started:A. as soon as the patient’s neurological and medical condition stabilizesB. after 2 weeks of onset neurologic symptomsC. as soon as the diagnosis of a stroke is madeD. as soon as a motor grade of at least 2/5 in the affected extremities are noted

31.This reflex is integrated in the spinal cordA. Asymmetric tonic neck reflexB. Optical rightingC. Flexor withdrawalD. Landau reflex

32. This reflex develops mouth opening, helps find the breast and develops various tongue position

A. Sucking reflexB. Rooting reflexC. Automatic head turningD. Demand feeding reflex

33.Asymmetric Tonic Neck reflexA. Enables each side of the body to be used separatelyB. Coordinates with tonic labyrinthine reflex for turning the bodyC. Integrated in the cortexD. Appears at six month of age

34. At four months of age the child is expected toA. Sit with good balanceB. Grasp objects dangled in front of faceC. Increased flexion of the trunkD. Rolls over from supine

35. A child of 3 years is expected to A. Roller skatesB. Dresses without supervisionC. Walks up stairs alternating feetD. Skips

36. A dyskenitic Cerebral palsy is frequently associated with A. Bilirubin encephalopathyB. Anoxic spellsC. Birth TraumaD. Hypotonic type

37.True of a geriatric individualA. Most of them are institutionalizeB. They are not interested in sexual activities

Page 81: Surgery- board exam questions

C. They prefer to be independentD. They wanted to live with adult children

38.Some of the changes in the elderly isA. Decline in the number of motor unitsB. Increase in muscle massC. Preservation of vibratory perceptionD. Increase in visual acuity

39.Crystallized intelligence is preserved in elderly this isA. JudgmentB. Abstract thinkingC. comprehension D. Reasoning

40. Elderly has tendency to fall. One reason is a sudden drop of blood pressure which can be due to

A. Failure to take antihypertensiveB. Sudden change in position or abrupt standingC. Hearing lossD. Visual loss

41.Exercise prescription for an osteoporotic patient isA. SwimmingB. JoggingC. WalkingD. Bicycling

42.Osteoporosis has an earlier onset in women . And this is usually evident duringA. 1 to 3 years after menopauseB. 4 to 6 years after menopauseC. 5 to 7 years after menopauseD. 10 years after menopause

43.Elderly patient is prone to bed sore because ofA. Loss of subcutaneous fatB. More prominent blood vesselsC. DementiaD. Increased heat dissipation

44. Cardiorespiratory changes in elderlyA. Lower maximal heart rateB. Lower total lung capacityC. Less Reserve volumeD. Less cardiac output

45.To prevent hypotension in elderly it is best to observeA. Restriction of salt intakeB. Keep him in bed most of the timeC. Standing slowly while holding to a barD. Exercise upon standing

46. A spinal cord injury with weaker upper extremities than lower extremities isA. A posterior cord syndromeB. Central cord syndromeC. A bilateral brachial plexus injuryD. A hemi section of the spinal cord

Page 82: Surgery- board exam questions

47. An SCI patient states that he has intact sensation up to the umbilicus, without movement over his legs. This means

A. His sensory level is T10B. His corresponding motor level is T12C. His lesion is over T10 vertebraD. He is a quadriplegic

48.The most common site of heterotrophic ossification among burn patient is at the A. HipB. KneeC. ElbowD. Shoulder

49.A bedridden quadriplegic has painful inflamed thigh . The working diagnosis is DVT , it is best to differentiate this with

A. Fracture of the hipB. Heterotopic ossificationC. OsteomylitisD. TB arthritis

50.The most common tumor of the bone A. OsteosarcomaB. Multiple MyelomaC. MetastaticD. Osteod osteoma

51.The most common cause of Traumatic brain injury isA. Vehicular accidentB. FallC. Gunshot woundD. Sports injury

52.A C-curve scoliosis is common inA. Idiopathic scoliosisB. Scoliosis secondary to leg length discrepancyC. Scoliosis due to hemivertebraD. Paralytic scoliosis

53.Physical finding in scoliosis on convexity sideA. Prominent front chest B. Rotation of the spineC. Lower shoulder levelD. Prominent scapula

54. .Physical finding in scoliosis on concavity sideA. Rotation of the vertebral bodyB. Prominent front chestC. Wider rib SpaceD. Better chest excursion

55.. This syndrome of massive sympathetic discharge associated with the SCI patients with lesions at T6 level, characterized by headache, hypertension, diaphoresis and reflex bradycardia is known as:

A. areflexia C. autonomic dysreflexiaB. sacral sparing D. cauda equina

56. This is the most important aspect to consider in the management of decubitus ulcer:

Page 83: Surgery- board exam questions

A. relief of pressure C. use of hot packsB. use of transcutaneous nerve stimulation D. muscle strengthening

57. What is the key muscle for testing C7 myotome, according to the American Spinal Injury Association?

A. triceps C. bicepsB. deltoids D. first dorsal interossei

58. . This level is generally the highest level of injury at which spontaneous ventilation can be sustained. Injuries above this level generally require mechanical ventilation:

A. C4 C. C6B. C5 D. C7

59. This is referred to as an ectopic bone formation occurring within 6 months after spinal cord injury:

A. heterotopic ossification C. osteoporosisB. degenerative joint disease D. compression deformities of the spine

60. This is generally considered the drug of choice for spasticity in SCI:A. Diazepam C. Dantrolene sodiumB. Baclofen D. Clonidine

61. The most commonly observed Cumulative Trauma disorder isA. De QuervainsB. Trigger fingerC. Carpal tunnel syndromeD. Tennis elbow

62. The ff. is an indirect cost of injury among workersA. medical CostB. Lost Time WagesC. Training replacementsD. Compensation Premiums

63.The ff. is an indication for surgical release of CTSA. Severe causalgia B. Weak Abductor Pollicis brevisC. Atrophy of hypothenar eminenceD. edema on the wrist

64. The ff. is a feature of Impingement Syndrome A. Limited Passive ROM of the shoulderB. Pain active on shoulder flexionC. Limited active abduction of the shoulderD. Positive Spurling sign

65. In Reflex sympathetic dystrophy the ff. is trueA. Pathology originates at the fingersB. Autonomic changesC. Complete shoulder ROMD. Splinting the shoulder helps a lot

66. The most common lump on the hand isA. Trigger fingerB. Ganglion cystC. Tendenitis

Page 84: Surgery- board exam questions

D. Dupuytrens contracture

67. The Following can lead to Frozen Shoulder EXCEPTA. Rotator cuff TearB. Bicipital tendenitisC. Carpal Tunnel SyndromeD. Trigger finger

68. Avascular necrosis of the LunateA. PannersB. KeinbocksC. Dupuytrens D. Osgood

69. A worker comes with a lump on the dorsal elbow, it is movable, soft with tenderness. He has a habit on putting his weight at the elbow , he has A. Golfer’s elbowB. Tennis elbowC. TendinitisD. Student’s elbow

70. The best thing to do in acute tendenitis isA. To stretch it to increase tensile strengthB. To rest it in a splint C. To move it to prevent contractureD. To apply hot moist pack

71. The grip strength of a dominant hand is usually stronger than the non dominant hand by A. 10 lb. C. 30 lbsB. 20 lb. D. 40 lbs

72. A functional non organic sensation deficit is evident if there isA. vibratory deficit in the upper armB. sensory deficit in the whole leg C. numbness of the thumbD. numbness on the right side of the face

73. To prevent overvaluation and subjection to potential risk of tests. The following will suggest a non organic findings in back painA. Pain on straight leg raising at 30 degreesB. light pinch tenderness of skin on vast area of the backC. Sensory deficit on the lateral thighD. weak extensor hallucis longus

74. The most common type of muscular dystrophy isA. Limb Girdle B. DuschenneC. Facioscapulo-humeralD. Beckers

75. Cramps is the usual complain ofA. DuschenneB. MyotonicC. SMAD. Limb Girdle

76. Spinal dysraphism can be detected in utero after 18 weeks by taking the

Page 85: Surgery- board exam questions

A. Amniotic C -reactive proteinB. Amniotic Alpha feto proteinC. Amniotic phosphofructokinaseD. Amniotic amylase

77. A positive Gowers sign is A. Pathognomonic in muscular dystrophyB. Present in pelvic and proximal leg weaknessC. seen in hip flexion contracture D. a feature of McArdle’s disease

78. Ischemic Compression Test is for A. Duschenne MDB. MyotoniaC. SMAD. McArdle’s Disease

79. A dive bomber sound on EMG is seen inA. Beckers MDB. MyotoniaC. Spinal Dysraphism D. SMA

80. Fasciculation’s of the tongue and limb muscles is frequently seen in A. Beckers MD C. Spinal DysraphismB. Myotonia D. SMA

81. Most of children with this condition cannot reach adulthood exceptA. DuschenneB. Fascioscapulohumeral dystrophyC. Werdnig Hoffman (Acute)D. Pompe’s Disease

82. The most common cause of death of children with myopathies isA. Renal Problem C. Pulmonary problem B. Weakness D. Myocardial infarction

83. This reflex is integrated in the spinal cordA. Asymmetric tonic neck reflexB. Optical rightingC. Flexor withdrawalD. Landau reflex

84. This reflex develops mouth opening, helps find the breast and develops various tongue position

A. Sucking reflexB. Rooting reflexC. Automatic head turningD. Demand feeding reflex

85. Asymmetric Tonic Neck reflexA. Enables each side of the body to be used separatelyB. Coordinates with tonic labyrinthine reflex for turning the bodyC. Integrated in the cortexD. Appears at six month of age

Page 86: Surgery- board exam questions

86. Patient with Spina bifida should be examined forA. Presence of strabismusB. Presence of Mongolian spotC. Presence of hip dislocationD. Presence of hormonal problem

87. . This instrument is used to measure range of motion of joints:A. tape measure C. tachometerB. goniometer D. planometer

88. . Which of the following is considered as a deep heating modality:A. heating pads C. ultrasoundB. whirlpool baths D. paraffin baths

89. On stimulation the baby extend and abduct the limbs followed by flexion and adductionA. If this is a normal baby he is less than four monthsB. This is a reflex integrated in the midbrainC. This reflex will not interfere in balance and rolling overD. This is a normal reaction in baby of all ages

90. . Stroke rehabilitation may be started:A. as soon as the patient’s neurological and medical condition stabilizesB. after 2 weeks of onset neurologic symptomsC. as soon as the diagnosis of a stroke is madeD. as soon as a motor grade of at least 2/5 in the affected extremities are noted

91.This reflex is integrated in the spinal cordA. Asymmetric tonic neck reflexB. Optical rightingC. Flexor withdrawalD. Landau reflex

92. This reflex develops mouth opening, helps find the breast and develops various tongue position

A. Sucking reflexB. Rooting reflexC. Automatic head turningD. Demand feeding reflex

93.Asymmetric Tonic Neck reflexA. Enables each side of the body to be used separatelyB. Coordinates with tonic labyrinthine reflex for turning the bodyC. Integrated in the cortexD. Appears at six month of age

94. At four months of age the child is expected toA. Sit with good balanceB. Grasp objects dangled in front of faceC. Increased flexion of the trunkD. Rolls over from supine

95. A child of 3 years is expected to A. Roller skatesB. Dresses without supervisionC. Walks up stairs alternating feetD. Skips

Page 87: Surgery- board exam questions

96. A dyskenitic Cerebral palsy is frequently associated with A. Bilirubin encephalopathyB. Anoxic spellsC. Birth TraumaD. Hypotonic type

97.True of a geriatric individualA. Most of them are institutionalizeB. They are not interested in sexual activitiesC. They prefer to be independentD. They wanted to live with adult children

98.Some of the changes in the elderly isA. Decline in the number of motor unitsB. Increase in muscle massC. Preservation of vibratory perceptionD. Increase in visual acuity

99.Crystallized intelligence is preserved in elderly this is

A. JudgmentB. Abstract thinkingC. comprehension D. Reasoning

100. Elderly has tendency to fall. One reason is a sudden drop of blood pressure which can be due to

A. Failure to take antihypertensiveB. Sudden change in position or abrupt standingC. Hearing lossD. Visual loss

101.Exercise prescription for an osteoporotic patient isA. SwimmingB. JoggingC. WalkingD. Bicycling

102.Osteoporosis has an earlier onset in women . And this is usually evident duringA. 1 to 3 years after menopauseB. 4 to 6 years after menopauseC. 5 to 7 years after menopauseD. 10 years after menopause

103.Elderly patient is prone to bed sore because ofA. Loss of subcutaneous fatB. More prominent blood vesselsC. DementiaD. Increased heat dissipation

104. Cardiorespiratory changes in elderlyA. Lower maximal heart rateB. Lower total lung capacityC. Less Reserve volumeD. Less cardiac output

105.To prevent hypotension in elderly it is best to observeA. Restriction of salt intake

Page 88: Surgery- board exam questions

B. Keep him in bed most of the timeC. Standing slowly while holding to a barD. Exercise upon standing

106. A spinal cord injury with weaker upper extremities than lower extremities isA. A posterior cord syndromeB. Central cord syndromeC. A bilateral brachial plexus injuryD. A hemi section of the spinal cord

107. An SCI patient states that he has intact sensation up to the umbilicus, without movement over his legs. This means

A. His sensory level is T10B. His corresponding motor level is T12C. His lesion is over T10 vertebraD. He is a quadriplegic

108.The most common site of heterotrophic ossification among burn patient is at the A. HipB. KneeC. ElbowD. Shoulder

109.A bedridden quadriplegic has painful inflamed thigh . The working diagnosis is DVT , it is best to differentiate this with

A. Fracture of the hipB. Heterotopic ossificationC. OsteomylitisD. TB arthritis

110.The most common tumor of the bone A. OsteosarcomaB. Multiple MyelomaC. MetastaticD. Osteod osteoma

111.The most common cause of Traumatic brain injury isA. Vehicular accidentB. FallC. Gunshot woundD. Sports injury

112.A C-curve scoliosis is common inA. Idiopathic scoliosisB. Scoliosis secondary to leg length discrepancyC. Scoliosis due to hemivertebraD. Paralytic scoliosis

113.Physical finding in scoliosis on convexity sideA. Prominent front chest B. Rotation of the spineC. Lower shoulder levelD. Prominent scapula

114. .Physical finding in scoliosis on concavity sideA. Rotation of the vertebral bodyB. Prominent front chest

Page 89: Surgery- board exam questions

C. Wider rib SpaceD. Better chest excursion

115.. This syndrome of massive sympathetic discharge associated with the SCI patients with lesions at T6 level, characterized by headache, hypertension, diaphoresis and reflex bradycardia is known as:

A. areflexia C. autonomic dysreflexiaB. sacral sparing D. cauda equina

116. This is the most important aspect to consider in the management of decubitus ulcer:A. relief of pressure C. use of hot packsB. use of transcutaneous nerve stimulation D. muscle strengthening

117. What is the key muscle for testing C7 myotome, according to the American Spinal Injury Association?

A. triceps C. bicepsB. deltoids D. first dorsal interossei

118. . This level is generally the highest level of injury at which spontaneous ventilation can be sustained. Injuries above this level generally require mechanical ventilation:

A. C4 C. C6B. C5 D. C7

119. This is referred to as an ectopic bone formation occurring within 6 months after spinal cord injury:

A. heterotopic ossification C. osteoporosisB. degenerative joint disease D. compression deformities of the spine

120. This is generally considered the drug of choice for spasticity in SCI:A. Diazepam C. Dantrolene sodiumB. Baclofen D. Clonidine

_____ 1. A female cancer patient is in constant pain which is severe, radiating, shooting and electric like in character. Most likely cause of her pain is:A. The enlarged mass itselfB. Compression of a nerve by the enlarged massC. Bone metastasisD. Treatment related

_____ 2. Gate’s Control Theory of pain is the basis for all the following non pharmacologic pain management EXCEPT:A. Transcutaneous electrical nerve stimulation C. Cold compressB. Massage D. Biofeedback

_____ 3. A peripheral type of pain:A. Myofascial pain syndrome C. Post herpetic neuralgiaB. Reflex sympathetic dystrophy D. Tension headache

_____ 4. Management of pain due to excision of mass in the anterior abdominal wall:A. Non-steroidal anti inflammatory agents (NSAIDs)B. OpioidsC. Opioids and NSAIDsD. NSAIDs, opioids plus anti depressants

_____ 5. Chronic use or prolonged intermittent administration of this opioids. Results in neurotoxicity due to its metabolite.

Page 90: Surgery- board exam questions

A. Meperidine C. FentanylB. Morphine D. Oxycodone

_____ 6. Major toxicity of short term use of NSAIDs:A. Exacerbation of hypertension C. Gastro duodenal irritationB. Hemostasis D. Acute deterioration of renal function

_____ 7. Effective in neuropathic pains:A. Opioids C. AntidepressantsB. NSAIDs D. Anticonvulsants

_____ 8. Acupuncture based on traditional Chinese medicine.A. Solid needles inserted into trigger bondsB. Solid needles inserted along meridiansC. Local anesthetic injected into or near trigger bondsD. Electrical stimulation along trigger bonds

_____ 9. A focussed concentration used as a guide to patients to focuss away from their pain.A. Biofeedback C. AromatherapyB. Hypnosis D. Relaxation exercises

_____ 10. Which of the following NSAIDs act on the central cyclooxygenase?A. Ibuprofen C. AspirinB. Paracetamol D. All of the above

_____ 11. The gold standard to which all of the other opioids are being compared to.A. Demerol C. NalbuphineB. Morphine D. Fentanyl

_____ 12. Which of the following opioids is an agonist-antagonist.A. Demerol C. NalbuphineB. Morphine D. Fentanyl

_____ 13. Mechanism of action of ketamine.A. Inhibits adenyl cyclase in the post-synaptic nerveB. Enhances GABA receptor in the pre-synaptic nerveC. Prevents the removal of magnesium from the NMDA receptor in the post-

synaptic nerveD. Prevent substance P. from occupying NKI receptor in the post synaptic cell

membrane.

_____ 14. Opioids receptor responsible for the respiratory depressant effect of opioidsA. Mu 1 receptor C. Kappa 1 receptorB. Mu 2 receptor D. Kappa 2 receptor

_____ 15. The following local anesthesic exhibit vasoconstricting effect at low and high doses.A. Lidocaine C. RopivacaineB. Bupivacaine D. Cocaine

_____ 16. One of the following is on objective sign of CNS toxicity:A.Convulsion C. DrowsinessB. Dizziness D. Lightheadedness

_____ 17. Local anesthetic that belongs to amide group.A. Procaine C. Cocaine

Page 91: Surgery- board exam questions

B. Lidocaine D. Tetracaine

_____ 18. Local anesthetic that is converted to O-toluidine in the liver:A. Procaine C. PrilocaineB. Etidocaine D. Cocaine

_____ 19. Local anesthetic with arrtiarrhythmic effectA. Tetracaine C. LidocaineB. Bupivacaine D. Etidocaine

_____ 20. Addition of vasoconstrictors to local anesthetics will:A. shorten the duration of action of local anestheticB. Prolong the duration of action of local anestheticC. Increase the potency of the drugD. Decrease the potency of the drug

_____ 21. Local anesthetic that causes unidirectional block and re-entry type of cardiac arrhythmia.A. Bupivacaine C. LidocaineB. Etidocaine D. Tetracain

_____ 22. The median nerve originates from the:A. Lateral cord C. Both A & BB. Medial cord D. Neither A & B

_____ 23. The Dorsalis Pedis Artery is used as landmark to block the:A. Superficial Peroneal nerve C. Deep Peroneal nerveB. Saphenous nerve D. Sural nerve

_____ 24. The nerve blocked, when local anesthetic is injected between the palmaris longus and tendon longus and flexor carpi radialis at the wrist.A. Ulnar nerve C. Radial nerveB. Median nerve D. None of the above

_____ 25. Caudal anesthesia is a form of:A. Spinal anesthesia C. Local blockB. Peripheral block D. Epidural block

_____ 26. Boundaries of the anatomic Snuff Box EXCEPT:A. Extension Pollicis Longus C. Extension Pollicis BrevisB. Abduction Pollicis Longus D. Abductor Pollicis Brevis

_____ 27. The nerve blocked posterior to the lateral malleolus.A. Sural nerve C. Saphenous nerveB. Tibial nerve D. Superficial peroneal nerve

_____ 28. Complication of pudendal nerve block during vaginal delivery:A. Inadvertent IV injection C. Puncture of fetal headB. Injury to the nerve D. Toxicity to local anesthetic

_____ 29. Delayed complication of spinal anesthesiaA. Difficulty of breathing C. HypotensionB. Urinary retention D. Nausea

_____ 30. The nerve found at the anatomic snuff box nerveA. Median Nerve C. Radial nerveB. Ulnar Nerve D. Digital Nerve

Page 92: Surgery- board exam questions

_____ 31. Nerve blocked in Retrobulbar nerve blockA. Occulomotor nerve C. Ophthalmic nerveB. Ciliary Ganglion D. Optic nerve

_____ 32. Branch of intercostals nerve often spared when injection of local anesthetic is done at the anterior axillary line:A. Anterior cutaneous branch C. Posterior cutaneous branchB. Medial cutaneous branch D. Lateral cutaneous branch

_____ 33. Landmark in doing superficial cervical plexus block.A. Internal jugular vein C. Anterior scalene muscleB. Sternocleidonastoid muscle D. Omohyoid muscle

_____ 34. Approach in doing brachial plexus block that anesthetize the entire plexus:A. Supraclavicular Approach C. Interscalene ApproachB. Infraclavicular Approach D. Axillary Approach

_____ 35. Post spinal headache as a delayed complication of spinal anesthesia is due to:A. Hamatoma C. Muscle strainB. CSF leakage D. Referred pain

_____ 36. T6 level of sensory blockade is at the level of :A. Nipple line C. UmbiliusB. Subcostal line D. Xiphoid process

_____ 37. The line joining the highest points of iliac crest (ASIS) crosses:A. L1 interspace C. L4 interspaceB. L3 interspace D. L2 interspace

_____ 38. In adult the spinal cord ends at the level of:A. S1 C. L1B. S2 D. L3

_____ 39. Drug administration that allows patient to titrate analgesics according to their needs:A. Patient controlled analgesia (PCA)B. Intravenously on a per need basicC. Transdermal therapeutic systemD. Oral on a time contingent basis

_____ 40. A patient can be discharged from the Postoperative care unit if he is:A. Drowsy but responds to painB. Blood pressure is more than 30% of baselineC. Still in pain even if analgesics has been givenD. Pulse oximeter reading is 97% at room air.

CASE: LA, 35 years old female is schedule for appendectomy:BP=130/80 PR=88/min anesthesia given is subarachnoid block:

_____ 41. What is the ideal level of the block?A. High spinal C. Low spinalB. Midspinal D. Saddle block

_____ 42. Upon doing the pinprick test, the sensory block is at the level of the nipple (T4). Give the level of motor block:A. T6 C. T2

Page 93: Surgery- board exam questions

B. T8 D. T10

_____ 43. Five minutes after assuming the supine position; the BP because 90/60 PR=50/min Give the reason for hypotension:A. Para sympathetic block C. Sympathetic blockB. Motor block D. Sensory block

_____ 44. What is the immediate management? Of hypotensionA. Head up position C. oxygenationB. Increase IV fluid infusion D. Give vasoconstrictors

_____ 45. What is the immediate management of the bradycardia?A. Ephedrine sulfate IV C. Intravenous fluid infusionB. Atropine sulfate D. Midazolam IV

_____ 46. Intake of an agent by the body and the uptake of the agent by the tissues is termed.A. Circulation C. AbsorptionB. Elimination D. Metabolism

_____ 47. Distribution of general anesthetic drugs is influenced by:A. Route of administration C. BioavailabilityB. Regional blood flow D. Metabolites

_____ 48. Discovered oxygen and nitrous oxide:A. John snow C. Crawford T. LongB. Joseph Priestly D. Henry Hickman

_____ 49. Muscle relaxation is the result of :A. Mental block C. Motor blockB. Sensory block D. Block of reflexes

_____ 50. True of nondepolarizing muscle relaxant:A. Competetive inhibition with acetylcholineB. Prolonged depolarizationC. Reduced sensitivity to post junctional membraneD. Action is reversed by acetylcholinesterases

_____ 51. Last CNS structure depressed by general anesthesia is:A. Cortical center C. Medullary centerB. Spinal cord D. Basal ganglion & cerebellum

_____ 52. A 16 year old healthy patient with incarcerated inguinal hernia, for inguinal herviorhaphy the American Society of Anesthesiologists risks classification under:A. Class I C. Class IIIB. Class II D. Class IV

_____ 53. Cardiovascular intolerance to changes in position after anesthesia is a sign of:A. Insufficient depth of anesthesia C. Excessive depth of anesthesiaB. Sufficient depth of anesthesia D. All of above

_____ 54. Heat and cold sensations are transmitted through this type of nerve fibers:A. C- Fibers C. A beta fibersB. A- Delta fibers D. All of the above

Page 94: Surgery- board exam questions

_____ 55. Sharp cut on the skin are transmitted mostly through this type of nerve fibers:A. C-fibers C. A beta fibersB. A delta fibers D. All of the above

_____ 56. Referred pain occurs due to convergence of nerve fibers in this neurons:A. Nociceptive specific neurons C. Brain stem neuronsB. Wide dynamic range neurons D. All of the above

_____ 57. Which of the following is an excitatory nerve transmitter for the pain pathway:A. Glutamate C. AcetylcholineB. Serotonin D. Norepinepherine

_____ 58. In the dorsal horn of the spinal cord, the opioid receptor are found in:A. Lamina I C. Lamina IIIB. Lamina II D. Lamina V

_____ 59. First order neuron are found in:A. Dorsal horn C. ThalamusB. Ventral root ganglion D. Dorsal root ganglion

_____ 60. Primary afferent fibers with fastest conduction velocity:A. C type fiber C. A beta fibersA delta nerve fiber D. D. fibers

_____ 61. Known as carrier gas:A. Nitrous oxide C. Carbon dioxideB. Oxygen D. Carbogen

_____ 62. True of ketamine HCL effect:A. Lower intracranial pressure C. Increase intraocular pressureB. Lower blood pressure D. Increase awareness

_____ 63. Which of the following is a belladona alkaloid?A. Diphenhydramine HCL C. Morphine sulfateB. Atropine sulfate D. Nalbuphine

_____ 64. An inherited abnormality resulting from exposure of patient to certain anesthesic:A. Malignant hypothermia C. Benigs hyperthermiaB. Malignant hyperthermia D. Benigs hypothermia

_____ 65. One of the following is a weak anesthetic but a potent analgesic.A. Nitrous oxide C. OxygenB. Carbon dioxide D. Ethyline

_____ 66. Local anesthetic with vasoconstriction effect:A. Lidocaine C. PontocaineB. Bupivacaine D. Cocaine

_____ 67. Systemic toxicity to local anesthesia is manisfested by:A. Convulsions C. MethemoglobinemiaB. Skeletal muscle irritation D. Addiction

_____ 68. Causes methemoglobinemia because it is degraded in the liver to otoluidine w/c causes oxidation of hemoglobine.A. Xylocaine C. PrilocaineB. Coccaine D. Bupivacaine

Page 95: Surgery- board exam questions

_____ 69. Factors that affect absorption of local anesthetic.A. Site of injection C. pKAB. Protein buiding D. pCO2

______ 70. Duration of action of local anesthetic correlates with :A. Lipid solubility C. pKAB. Protein buiding D. Degree of Ionization

____ 71. Potency of a local anesthetics correlates with.A. Lipid solubility C. PKAB. Protein buiding D. Degree of Ionization

_____ 72. Onset of action of a local anesthetic correlates with.A. Lipid solubility C. pKAB. Protein buiding D. Degree of Ionization

_____ 73. When asked about the pain he categorically says it is minimal. Numerical score is.A. 1 to 3 C. 8 to 10B. 5 to 6 D. zero

_____ 74. Tramadol and Meperidine should not be given in patients receiving.A. Adenycyclase inhibitor C. Lipooxygenase inhibitorB. Cyclioxygenase inhibitor D. Monoamineoxydase inhibitor

_____ 75. Mechanism of action of tramadol.A. A weak mu receptor agonistB. Inhibits nor adrenalin receptorC. Inhibits serotonin receptorD. All of the above

_____ 76. Mechanism of action of NSAIDsA. Inhibition of lipooxygenase C. Inhibition of phospholipaseB. Inhibition of cyclioxygenase inhibitor D. All of the above

_____ 77. Common complication of retrobulbar block in extremely myopic eye.A. Globe perforation C. Paralysis of retrobulbar nerveB. Retrobulbar hemorrhage D. Blindness

_____ 78. The landmark for doing deep peroneal nerve block.A. Dorsalis pedis vein C. Dorsalis pedis arteryB. Medial malleolus D. Lateral malleolus

_____ 79. Most common complication of intercostals nerve block.A. Pneumothorax C. ConvulsionsB. Intravascular injection D. Failed block

_____ 80. Local anesthetic technique which uses lidocaine spray 10%.A. Field block C. Bier blockB. Infiltration D. Topical

_____ 82. Needle puncture during spinal anesthesia can be made safely at the interspace.A. Between L1 – L2 C. L4 – L5B. T12 – L1 D. L5 – S1

Page 96: Surgery- board exam questions

_____ 83. Mechanism of chronic pain which is often referred to as “nociceptive pain”.A. Peripheral C. Central peripheralB. Central D. Psychophysiologic

_____ 84. Management for central type pf pain:A. NSAIDs C. AspirinB. Opioids D. Steroids

_____ 85. Nerve missed in doing the axillary approach in brachial plexus blockA. Axillary nerve C. Ulnar nerveB. Lateral rectoral nerve D. Musculo cutaneous nerve

_____ 86. Nerve blocked posterior to the medial malleolus.A. Sural nerve C. Superficial peroneal nerveB. Tibial nerve D. Saphenous nerve

_____ 87 . The anesthetic potency of volatile liquid anesthetic is measured by:A. Minimum arterial concentrationB. Minimum alveolar concentrationC. Maximum alveolar concentrationD. Maximum airway concentration

_____ 89. The most common cause of airway obstruction is general anesthesia is:A. Salivary secretions C. Presence of enlarged adenoidsB. Falling back of the tongue D. Enlarged tonsils

_____ 90. Green is the standard color code for :A. Carbon dioxide C. CarbogenB. Oxygen D. Nitrous oxide

_____ 91. Elimination of general inhalation anesthetics is mainly thru:A. Kidney C. BloodB. Liver D. Lungs

_____ 92. Part of the anesthesia machine that releases gas to the atmosphere.A. Vaporizer C. Pop-off valueB. Pressure reducing value D. Flow meter

_____ 93. A patient with severe systemic disease that is a constant threat to life, will have a physical status classification.A. ASA I C. ASA IIIB. ASA II D. ASA IV

_____ 94. Lowest intensity at which a given stimulus is perceived as painful.A. Pain tolerance C. Pain sufferingB. Pain threshold D. All of the above

_____ 95. One of the following is an example of acute pain.A. Arthralgia C. HeadacheB. Low back pain D. Post-operative pain

_____ 96. This gas is known as a “Laughing Gas”A. Carbon dioxide C. Nitrous oxideB. Ethylene D. Oxygen

_____ 97. Type of ventilation used for apneic patient.A. Spontaneous ventilation C. Assisted ventilation

Page 97: Surgery- board exam questions

B. Controlled ventilation D. Positive end expiratory pressure ventilation

_____ 98. Transduction occurs in the :A. Peripheral terminals of primary afferent neurons B. Projections of spinal neuronsC. Properties of diencephalic neuronsD. Projections of wide dynamic range neurons

_____ 99. Characteristics of acute pain:A. Generate reflexes C. Long duration intervals for monthsB. Associated with depression D. Recurs at intervals for months

_____ 100. The nerve fiber is freely permeable to:A. CL – ions C. Na + ionsB. K + ions D. Ca + ions

B 2 Which of the following layers of the retina does not terminate at the optic disc 23-24margin:A. Retinal pigment epitheliumB. Nerve fiber layerC. Ganglion cell layerD. Layer of rods and cones

C 3 The adult orbital measurement is attained at this age: 35A. 5th yearB. 3rd yearC. 9th yearD. 20th year

A 4 The orbital space formed by the recti muscles and their intermuscular membrane 36with the Tenon’s capsule:A. Central surgical spaceB. Subperiosteal spaceC. Episcleral spaceD. Peripheral surgical space

C 5 The condensation of fibrous tissues that thickens to form a sling (hammock) upon 39which the globe rests:A. Whitnall’s ligament B. Check ligamentC. Ligament of LockwoodD. Annulus of Zinn

D 6 Which of the following does not pass thru the Superior orbital fissure: 37A. AbducensB. TrochlearC. OculomotorD. Optic nerve

B 7 The extrinsic muscle that is not inserted in front of the equator: 40A. Medial rectusB. Superior rectusC. Lateral rectusD. Inferior rectus

A 8 The adult size of the cornea is attained at: 3A. 6th year of lifeB. at birthC. 3rd year of lifeD. 9th year of life

C 10 Which of the following is not TRUE of the Meibomian gland: 46A. Does not communicate with hair follicles

Page 98: Surgery- board exam questions

B. Responsible for secreting the oily layer of the tear filmC. Modified sweat glandD. Originates from the tarsus

B 20 True of the Optic nerve, except: 60-61A. Consists of about 1 million axons from the ganglion cells

of the retina.B. Emerges from the posterior aspect of the globe.C. Myelinated all throughout.D. None of the above

B 21 True of Conjunctival injection: 211-212A. Vessels most numerous at the limbus and fade toward

the fornixB. Vessels constrict with a drop of 1:1,000 epinephrineC. Associate with keratitis, iridocyclitis, and angle closure

glaucomaD. Cornea cloudy, pupils distorted, vision reduced

C 22 Marked enlargement of the optic disk is rarely seen, but its occurrence 353-354suggests:A. myopiaB. posterior staphylomaC. bothD. neither

A 33 Any opacity in the lens is called: 369A. CataractB. LeukomaC. NebulaD. None of the above

D 34 The optic nerve is actually a collection of axons of the: 353A. cells of the inner nuclear

layerB. cells of the outer nuclear

layerC. photoreceptorsD. ganglion cells

D 35 As it courses through the orbit, the optic nerve derives its blood supply from the: 352A. pial vesselsB. arachnoid vesselsC. vessels of the duraD. d. central retinal artery

B 36 The center of the visual field corresponds to visual perception from the 295

following anatomic structure:A. the optic nerveB. the foveaC. the ora serrataD. none of the above

C 37 Mass lesions of the pituitary gland classically produce the following field pattern: 531

A. quadrantic hemianopsiaB. binasal hemianopsiaC. bitemporal hemianopsiaD. centrocecal scotoma

A 38 To enable stereoscopic vision, the visual fibers are so arranged that fibers over 66-67

the nasal retina decussate while temporal fibers continue ipsilaterally to their corresponding lateral geniculate body. The crossover of fibers occur at the:

A. optic chiasmB. orbitC. cannaliculusD. none of the above

Page 99: Surgery- board exam questions

B 39 Despite the seemingly horrendous appearance of the optic disc in acute 357papilledema, vision remains fairly good and the only deficit in the visual field is a:

A. cecocentral scotomaB. enlarged blind spotC. bitemporal hemianopsiaD. central scotoma

B 41 The following is true of optic nerve meningiomas: 364A. occurs most frequently during the first 2 decades of lifeB. symptoms are different from that of sphenoid wing meningiomasC. visual loss mainly through compression or atrophyD. more symptomatic than gliomas

B 42 Ischemic optic neuritis presents with this classical field defect: 360A. enlarged blind spotB. cecocentral scotomaC. altitudinal field defectD. bitemporal hemianopsia

A 50 True of pinguecula: 214A. benign degenerative tissueB. tumor of the eyelidsC. more common in the youngD. all of the above

Page 100: Surgery- board exam questions

D 51 True of symblepharon: 215A. chief cause is chemical burnB. keratitis occurs because of exposure C. obliterate conjunctival cul-de-sacD. all of the above

C 74 Middle coat of the eye is composed of the following, except: 15-16A. IrisB. Ciliary bodyC. RetinaD. Choroid

D 75 Structure that regulates the amount of light reaching the visual receptors 20of the eye: A. CorneaB. AqueousC. VitreousD. Iris

B 76 The embryologic tertiary vitreous is known as what structure in the adult eye: 72A. Ora

serrataB. Vitreou

s bodyC. ZonulesD. None of

the above

A 90 Portion of the Tenon capsule that forma a sling upon which the globe rests: 39A. ligament of lackwoodB. lower tendon of zinnC. whitnall’s ligamentD. check ligament

OPHTHALMOLOGY QUESTIONS

Conjunctivitis1. The type of conjunctivitis characterized by a localized whitish nodule with a necrotic

excavated center surrounded by conjunctival injection:a. epidemic keratoconjunctivitisb. corneal ulcerc. phlyctenular conjunctivitis (Espiritu etal, p. 54) d. vernal conjunctivitis

Cataract2. The virus that can cause congenital cataract:

a. adenovirusb. picornavirusc. rubella (Newell, p. 377)

1

Page 101: Surgery- board exam questions

d. rubeola

Glaucoma

3. Carbonic anhydrase inhibitors are given as a treatment for glaucoma. Its mechanism of action is:

a. to increase the aqueous humor outflow facilityb. to decrease aqueous humor production (Espiritu, p. 101)c. to decrease production of aqueous and enhance its exitd. to increase aqueous humor production

Cornea4. While cutting grass in his garden, a 50-year old male felt a small piece enter his left eye.

He experienced foreign body sensation and eye redness. No consult was done and no medications were instilled. A few days later, he noted a fluffy, grayish white elevation on the center of his eye. Blurring of vision was also noted. Which of the following medications is contraindicated?

a. atropine sulfateb. 5% natamycinc. corticosteroids (Newell, p. 249)d. amphotericin B

5. Softening of the cornea associated with malnourishment:a. keratoconusb. keratomalacia (Espiritu etal, p. 60)c. Mooren’s ulcerd. Staphyloma

Retina6. An ophthalmic emergency characterized by sudden blurring or complete loss of vision

with a characteristic cherry-red spot on funduscopy:a. central retinal vein occlusionb. branched retinal artery occlusionc. branched retinal vein occlusiond. central retinal artery occlusion (Espiritu, p. 88)

7. A 45-year old female suddenly experienced blurring of vision, with a sensation of a curtain in a part of the field of vision. The primary consideration in this patient is:

a. diabetic retinopathyb. retinal detachment (Espiritu, p. 92)c. central serous chorio-retinopathyd. hypertensive retinopathy

8. A 2-year-old boy was brought in for consultation because the mother noted a “white pupil”. Which of the following is NOT a differential diagnosis for this case?

a. congenital glaucoma ( Newell, p. 326)b. retrolental fibroplasiasc. persistent hyperplastic primary vitreousd. toxocariasis

9. Which of the following is responsible for scotopic vision or dim illumination:a. rods only (Espiritu, p. 86)b. cones onlyc. both A & Bd. none of the above

Systemic Ophthalmology10. The ocular lesion in congenital toxoplasmosis is characterized as:

2

Page 102: Surgery- board exam questions

a. corneal dendritic ulcer which stains with fluorescein b. follicular conjunctivitis, optic neuritisand non-granulomatous uveitisc. single choroidoretinal scar prominently in the posterior fundus (Espiritu, p.126)d. big necrotic lesion in the retina and choroids surrounded by smaller lesions

Lacrimal apparatus11. A 45-year-old male presents with a painless mass on the upper outer portion of his upper

eyelid. There was moderate proptosis. Which is true of his condition?a. It is the second most common epithelial tumor of the lacrimal glandb. It has the worst prognosis c. It contains mesenchymal elements and double-layered tubular epithelial units

(Newell, p. 263)d. It is composed of aggregates of small undifferentiated neoplastic cells separated

by small and large cystoid spaces with mucin

Eyelids12. A chronic inflammatory lipogranuloma of a meibomian gland, characterized by a gradual

painless swelling of the gland without other external signs of inflammation:a. external hordeolumb. chalazion (Newell, p. 204)c. meibomianitisd. internal hordeolum

13. The palpebral fissure among Filipinos measures:a. 8-12 mm in height and 25-29 mm in lengthb. 9-12 mm in height and 26-29 mm in lengthc. 8-10 mm in height and 25-29 mm in lengthd. 8-10 mm in height and 26-29 mm in length ( Espiritu, p. 9)

Neuro-ophthalmology14. Passive swelling of the optic nerve occurring secondary to increased pressure in the

subarachnoid space of the meningeal coverings of the brain and optic nerves:a. papilledema (Newell, p. 357)b. pseudo-papilledemac. drusend. optic neuritis

3